NUR401 NIES, - exam 3

Ace your homework & exams now with Quizwiz!

Which of the following individuals is most likely to be the victim of violent crime?

15 year old black male

Community Mental Health Centers Act (CMHCA)

1964 legislation providing federal support for mental health services and measures to implement facilities to care for those who were mentally retarded and to construct community mental health centers. CMHCA mandated deinstitutionalization, or a halt to the long-held policy of keeping the severely mentally ill hospitalized. The intention was to reduce long-term care of seriously mentally ill persons by transferring treatment to the community

Which of the following problems depends on the responses of other people and therefore cannot be measured? a. Disability b. Handicap c. Impairment d. Injury

ANS: B In comparing these concepts, an impairment affects a human organ on a micro level; disability affects a person on an individual level; and a handicap involves society on a macro level of analysis. Table 21-1 points out that both impairment and disability may be objective and measurable, whereas handicap is neither, because it is an experience related to the responses of other people.

Which of the following best describes how many households with at least one child with a disabling condition would a school nurse working in an elementary school expect to see? a. About 5% b. About 10% c. About 15% d. About 20%

ANS: C About 15.1% of households with children have at least one child with a special health care need (disabling condition).

Which of the following groups is the fastest-growing component of the rural population? a. Disabled adults b. Elderly women c. Hispanic children d. Retirees

ANS: C Today, the proportion of Hispanic children is the fastest-growing component of the rural population, regardless of region.

A nurse established a new parent support group where new parents meet to share problems and solutions. After about 3 months, fewer people were attending. Which of the following actions should the nurse take next?

Accept that the new parents may now be comfortable in their role and no longer need a support group

What group of homeless individuals is known for their involvement in survival sex? Young adult gay men Middle-aged women Adolescents Elderly men

Adolescents Runaway or homeless adolescents make up a large percentage of all youth involved in prostitution. Many become involved because they need money to meet subsistence needs, hence the term survival sex. DIF: Cognitive level: Knowledge REF: Pages 438-439

HIV primarily affects women in which of the following ethnic populations? African Americans Hispanics Asians Caucasians

African Americans The HIV/AIDS epidemic represents a growing and persistent health threat to women in the United States, especially young women and African American women. HIV infection was the third leading cause of death for African American women ages 25 to 44 years and was among the fourth leading cause of death for those ages 45 to 54 years. HIV disease among Hispanic women aged 25 to 44 years was the fifth leading cause of death. DIF: Cognitive level: Knowledge REF: Page 48

Neuroendocrine and Neurochemical

Aging is result of changes in brain and endocrine glands.

42.) Which of the following describes the most important need of every school-aged child with a disability? Chapter 21 pg. 411

An individualized education plan (IEP)

Epidemiological data was also obtained during the community health assessment. Which of the following is most significant as a reflection of the health of both children and pregnant women? A. Teen pregnancy rates B. Infant mortality rates C. Preterm birth rates D. Immunization rates

B. Infant mortality rates Infant mortality rates are indicative of the health status of infants and children, as well as pregnant women. This rate is most significant because it is reflective of so many health-related factors, including maternal health, access to quality medical care, and so forth. This rate is commonly used as a point of comparison between countries when evaluating maternal-child health status.

20.) A client, accompanied by her son, spoke only Spanish while the nurse did not. Which of the following actions should be taken by the nurse to best meet the client's need? Chapter 13 pg. 233.

Call a Spanish interpreter employed by the agency.

22.) Which of the following is the most common cause of death among women after age 75?

Cardiovascular disease (CVD).

Two unrelated heterosexual adults that share a housing unit with their three children is an example of which of the following? Nuclear family Cohabitation Internal structure External structure

Cohabitation Cohabitation is defined as a living arrangement in which an unmarried couple live together in a long-term relationship that resembles a marriage. A nuclear family is composed of a husband, wife, and their immediate children. Internal and external structure of a family refer to the family and their relationships within the immediate family and the extended family. DIF: Cognitive level: Knowledge REF: Page 383

Which of the following best describes the documentation that the nurse should expect to complete during the home visit?

Completed Medicare forms per the Health Care Financing Administration (HCFA) regulations for reimbursement

Which of the following best describes why community health education programs are important?

It provides a way to reinforce voluntary behavior changes.

Which of the following factors is the single largest threat to child health? Chapter 16 slide# 30 pg. 301.

Poverty

A nurse was becoming very frustrated at the ongoing stream of farmworkers with respiratory problems each spring. Which of the following actions should be taken by the nurse?

Provide community education on herbicides and other chemicals spread each spring with information on self-protective measures, such as respirators.

What is the primary problem seen in Healthy People 2020's emphasis on choosing healthy lifestyle behaviors, such as daily exercise or healthy food choices?

Public policy emphasizes personal responsibility but ignores social and environmental changes needed for well-being.

Radiation Theories

Radiation may induce cellular mutations that promote aging.

Free Radical Theory

Reactive molecules containing extra electrical charges cause damage of protein, enzymes, and DNA. Results from normal metabolism, reaction with other free radicals, oxidation of ozone, pesticides.

A correctional nurse was preparing to give an inmate his psychotropic drug when the prisoner hit the nurse's arm, sending the medication flying, and yelled, "No more. I'm not taking that poison anymore!" The prisoner is obviously mentally ill. Which of the following actions should be taken by the nurse?

Recognize the prisoner's right to refuse treatment

Which of the following is a major challenge for health care provider education today?

Refocusing the curriculum from care of the individual to needs of aggregates

Which of the following are the most frequent problems suffered by the chronically homeless?

Severe mental illness and substance abuse disorders

A school nurse is conducting a program with high school students about adolescent sexuality. Which of the following information would the school nurse most likely include?

Teenage mothers are less likely to complete high school and often remain single and live in poverty.

Which of the following assumptions makes Orem's theory difficult to use?

That clients are able to control their environment

Annual Homeless Assessment Report (AHAR)

The U.S. Department of Housing and Urban Development (HUD) submits this report (AHAR) to Congress. It contains information about the total population and subpopulations in the U.S. experiencing homelessness.

Chronically homeless

The U.S. Department of Housing and Urban Development designates unaccompanied adults who are homeless for extended or numerous periods of time and have one or more disabling conditions as chronically homeless.

A new home health nurse realized that what was being observed and now being said by the client were very inconsistent with what had been documented while the client was hospitalized. Which of the following conclusions should be drawn by the nurse?

The amount of support available is different from what was stated earlier.

Using a public health perspective, which of the following has responsibility for a child's health? Chapter 16 slide# 34 pg. 306.

The entire community

Autoimmune Reactions

The immune response declines after young adulthood, body misidentifies aged cells and develops antibodies.

Which of the following best describes how the U.S. Census Bureau Surveys are helpful to a community health nurse?

They provide the distribution of age, sex, race, and ethnicity in the community.

Which of the following best describes when faith communities first began to be involved with health and healing?

Throughout history, faith communities provided basic health care

Which of the following best describes shoe leather epidemiology?

Traveling throughout the community to absorb an overall impression of it

A mother was debating whether or not to take her son to the clinic. His temperature was 104° F and rising so she gave him acetaminophen (Tylenol). Which of the following will most likely be the deciding factor in the mother's decision?

Whether or not the child is covered under insurance

Which of the following is consistent with the American Academy of Pediatrics (AAP) recommendations for breastfeeding?

a. A mother combines breastfeeding with the introduction of complementary foods until the infant is 1 year old

Which of the following groups of persons is most likely to be able to access health care?

a. Adults who work in large corporations

What have been the foci of international nursing research? (Select all that apply.)

a. Development of home care or transition from hospital to home b. Diverse clinical experiences c. Educational exchange programs for students and faculty

Which of the following is the most serious complication of women having osteoporosis?

a. Hip fracture

Which of the following best explains how the Public Health Service (PHS) Act, Title X, has been helpful to families?

a. It provides funds for family planning, including contraception.

glaucoma

an abnormal condition of elevated pressure within an eye caused by obstruction of the outflow of aqueous humor

A mother was concerned about her child dying of some childhood disease, because that is how some of her siblings died before her family came to the United States. Which of the following would be the most appropriate response by the nurse?

b. "Your child has been immunized, so the biggest danger is from accidents."

Which of the following statements best explains why parents and nurses are often concerned about adolescents?

b. Adolescents engage in very high-risk behaviors.

Which of the following is the greatest single cause of absenteeism from school and work among young women?

b. Dysmenorrhea

Based on theory, which of the following are men most likely to do when they become ill?

b. Ignore symptoms or refuse to see them as symptoms of illness and act tough

Which of the following actions would improve a man's probability of obtaining health screenings and health-promoting education?

b. Joining the military

Which single action has been most influential in changing men's health behaviors?

b. Legislative action

Which of the following statements best explains why men are sicker on hospital admission than women?

b. Men do not seek treatment until they are very ill.

preterm birth

birth before 37 completed weeks of gestation

hypertension

blood pressure of 140/90 mm Hg or greater, which significantly increases the risk of serious morbidity and mortality from coronary heart disease

A mother was very proud that she had cut back smoking from two packs a day, which she had smoked for years, to only half a pack a day. Which of the following would be the most appropriate response by the nurse?

c. "That must have been difficult, but you're probably still getting nicotine, so it's important you quit entirely."

A nurse has just been told by a friend that her physician has recommended a hysterectomy for fibroids in her womb. Which of the following would be the most appropriate response by the nurse?

c. "There are other alternatives. Why don't you seek a second opinion just to be sure a hysterectomy is best for you?"

Which of the following individuals is most likely to be the victim of violent crime?

c. A 15-year -old black male

a school nurse is trying to decrease the effects of an incredible health crisis attacking school children. Which of the following actions would be most appropriate for the nurse to take?

c. Establish recreational programs that encourage physical exercise each day

Which of the following groups of children is at the highest risk to have poor health?

c. Hispanic children living in the inner city

breast cancer

cancer found in the breast tissue; results in 15% of all cancer deaths

Which of the following groups would most need adult day care services?

caregivers who work all day

children with a disability (CWD)

children with special needs

Which is the most common sexually transmitted disease?

chlamydia

rural

communities having fewer than 20,000 residents or fewer than 99 persons per square mile

Which of the following entities has been most active in creating change and alternative approaches in health care delivery?

corporations

severe mental illness (SMI)

current or at any time during the past year having a diagnosable mental, behavioral, or emotional disorder with moderate, severe, or extreme functional behavior in specific areas of lifestyle

What proportion of U.S. citizens are children?

d. 24%

Which of the following statements is correct?

d. Males are more likely to be eligible for primary care services than females.

Which of the following best describes how the United Nations (UN) is able to implement its decisions?

d. Using its moral authority and world opinion

Which of the following theoretical approaches would be most useful to the nurse who wants to improve patient-provider communication and address the gap between what providers discuss and what families want to know about?

ecological framework

Which of the following describes a dimension of the social determinants of health (SDH) defined within Healthy People 2020?

education

people/persons with disabilities (PWD)

individuals living with special needs or disabilities

family interviewing

model that uses general systems and communication concepts to conceptualize health needs of families and a family assessment model to assess families' responses to "normative" events, such as birth or retirement, or to "paranormative" events, such as chronic illness or divorce

reasonable accommodations

modifications made that are necessary to allow a qualified person to participate in an educational program or employment setting at a level equal to that of a nondisabled person

Which of the following are the most common forms of elder abuse?

neglect

lead poisoning

occurs when lead builds up in the body, often over a period of months or years

mental health consumer

person who is under treatment for a psychiatric illness or disorder

child maltreatment

physical abuse, sexual abuse, emotional abuse, or neglect that occurs among children under the age of 18

Social Security Disability Insurance (SSDI)

receives disability trust fund monies (Social Security taxes paid by workers, employers, and self-employed workers)

instrumental functioning

routine activities of daily living

migrant farm workers

those who migrate to find work

ecomap

tool that is used to depict a family's linkages to their suprasystems. It portrays an overview of the family in their situation.

Family Health Assessment

used as a guide to assist the nurse in data collection and organization of the data collected from families over time

childhood immunization

vaccinations to protect children against several diseases that killed or disabled many children in the past

macular degeneration

A chronic eye disease that occurs when tissue in the macula, the part of the retina that is responsible for central vision, deteriorates

Which of the following disasters would most likely cause the most long-lasting mental health concerns for its victims?

A forest fire was deliberately set but was not a major concern until the fire suddenly changed direction and people attempting to evacuate at the last minute were burned in their cars.

A nurse carefully explained that the medication had to be taken three times a day, with each meal. The patient came to the clinic with symptoms of medication overdose. How should the nurse respond?

"Can you tell me when in the day you and your family eat?"

sexual harassment

"conduct of a sexual nature...unwelcome by the target...severe or pervasive enough to create an intimidating work environment" (Women Employed Institute, 1994)

As a community health nurse, Connie knows it is critical to consider the family in the larger context of the community and society. As Connie provides care to this family, she considers the typical family life cycle. This is consistent with an approach guided by which conceptual framework? A. Systems B. Developmental C. Structural-functional D. Crisis intervention

B. Developmental Consideration of "family life cycle" (the typical developmental path of a family) and "family development" (the individual, unique developmental path of a family) is consistent with the use of developmental theory

12.) A school nurse at a high school suggests that all students have an educational unit on newborn care in their required health class. Which of the following provides the best rationale for this suggestion? Chapter 16 pg. 303 slide# 32.

Because the first year of life is the most hazardous until old age.

A researcher is examining potential risk factors in comparison with disease at a specific time through collecting data regarding current exercise, sleep patterns, and current health status among 12-year-olds. Which of the following research studies would be most appropriate?

Cross sectional study

27.) Which of the following is the most likely reason for adult women to be injured? Chapter 17 pg. 329.

Domestic violence

5.) A woman vows to quit smoking and drinking alcohol immediately upon the surprising news that she is 2 months pregnant. Which of the following concerns would the nurse have for this woman? Chapter 16 pg. 290 "Preconception Health".

Fetal damage may already have occurred.

Programmed Theory of Aging

Humans are born with genetic program or biological clock that predetermines life span.

Which of the following best explains why health professionals and educators are stressing total abstinence of drugs instead of teaching appropriate social use?

If students perceive a drug as harmful, fewer actually try it.

Apoptosis

Programmed cell death that continuously occurs throughout life.

Which of the following statements best describes the relationship between the environment and health?

Research shows that a healthy environment increases quality of life and years of healthy living.

A scholar during the Sanitary Revolution created medical topographies. What was the advantage of these surveys?

Results demonstrated environmental factors related to regional disease.

Which of the following best explains why the rural population is growing when young persons from rural areas consistently are moving to urban areas to seek employment?

Retirees and other urbanites able to conduct business through telecommunication and travel are moving in.

Which of the following best describes how health care professionals are involved in the problem of homelessness?

Serious illnesses or disability creates high medical bills, which may lead to homelessness.

A nurse conducted a study of two skin lotions: (1) an inexpensive one and (2) an expensive one. The nurse found that there was a relative risk of 0.7 for skin sores using the inexpensive one and a relative risk of 1.2 using the expensive one. Which skin lotion should the nurse use?

The inexpensive lotion

Which of the following behaviors is a warning that a person is no longer just using a drugs recreationally but is abusing drugs and becoming addicted?

The person is less able to control frequency and amount of indulging.

A nurse is planning a health education program in the community. Which of the following considerations must be made by the nurse?

The sociopolitical cultural context of the community's environment

On the basis of all the work done by social scientists, approximately how many homeless persons should be expected and tolerated in our society?

There should not be homeless persons in America.

A nurse was assigned to create and implement a community education program with the goal of preventing diabetes among the Hispanic population. As the nurse did not speak Spanish and was just beginning to be comfortable with the neighborhood Hispanic culture, which of the following would be the best approach for the nurse to take?

Try to find Hispanic volunteers who may be able to do the actual teaching.

Which of the following statements is true concerning tuberculosis (TB) infections in the United States?

Under certain conditions, susceptible hosts may be infected by the TB bacillus.

Stochastic Theories

View aging as the result of random cellular damage that occurs over time. Cross-linking Theories

The mother confided in the nurse, "I don't think my child is improving." Which of the following statements would be the best response by the nurse?

What is causing your concern

A school nurse is developing a health education program for third grade students about how to play safely. Which of the following provides the best rationale for this nursing action? Chapter 16 pg. 296 (Toy-related injury). Slide# 28.

Without proper head protection, many sports can lead to traumatic brain injury TBI).

chronic condition

a condition that persists for at least 3 months or belongs to a group of conditions classified as chronic regardless of time of onset, such as tuberculosis, neoplasm, or arthritis

depression

a disorder characterized by an all-pervasive sadness that is present much of the time

anxiety disorder

a disorder involving tension within a person or system that increases emotional reactivity 19 crime any act that violates a law and may have criminal intent

anorexia nervosa

a disorder of self-starvation characterized by significant weight loss, amenorrhea, compulsive physical activity, preoccupation with food, and a distorted body image

acute condition

a type of illness or injury that usually lasts less than 3 months and either results in restricted activity or causes the patient to receive medical care 18 androgen any steroid hormone that increases male characteristics

aging

a universal human experience that culminates in a final end

genogram

a way to diagram the family for three generations of family members with the generally agreed-on symbols to denote genealogy

general systems theory

a way to explain how the family as a unit interacts with larger units outside the family and with smaller units inside the family

advance directive

a written document in which a competent person gives instructions about his or her health care that are to be implemented in the future if the individual is not able to make the decisions

Which of the following actions has been most effective in reducing HIV infections in some countries?

a. ABC campaign

A man has been off work because of a recent illness. Which of the following would be the most useful action for the employer to take to support this man?

a. Encourage the man to return to work as soon as possible, possibly at something less physically demanding

A man, who looked about 45 years old, gave a cashier at a grocery store the Special Supplemental Nutrition Program for Women, Infants, and Children (WIC) vouchers for some of the food he had just selected. Which of the following conclusions can be drawn from this scenario?

a. He had a teenage daughter who was pregnant or with a young child.

Which of the following best describes why a nurse working in a U.S. urban area should be knowledgeable about infectious diseases common to areas of Africa?

b. Every infectious disease is just one airplane ride away.

A woman vows to quit smoking and drinking immediately upon the surprising news that she is 2 months pregnant. Which of the following concerns would the nurse have for this woman?

b. Fetal damage may already have occurred.

Which of the following aspects of a developed country would be most worthwhile to be copied by a developing country?

b. Health care reform policies to improve health care delivery

Which gender perceives and reports having better health?

b. Males

stigma

associating negative qualities with having a mental illness

generalized anxiety disorder (GAD)

characterized by chronic, unrealistic, and exaggerated worry and tension about one or more life circumstances lasting 6 months or longer (APA, 2000). Symptoms of GAD include trembling, twitching, muscle tension, headaches, irritability, sweating or hot flashes, dyspnea, and nausea. Periods of increasing symptoms are usually associated with life stressors or impending difficulties.

Which of the following best explains how the Civil Rights Act amendments have been helpful to adult women who are employed?

d. Discrimination against pregnant women or conditions involving childbirth or pregnancy is prohibited.

Which of the following is one of the biggest threats faced by developing countries that are trying to grow economically and make a better life for their citizens?

d. Large population

Which of the following best explains why a community health nurse would choose to emphasize women's health? (Select all that apply.)

d. Many factors resulting in illness and death in women are preventable. e. Knowledge deficits prevent women from assuming responsibility for their own health. f. Women face barriers in accessing health care.

Which of the following outcomes for the infant and mother have occurred because of fetal monitoring?

d. No difference in outcomes for the infant but increased risk for the mother

A community health nurse suggested that a group of nursing students perform a diabetic screening at a location where groups of people meet. Which of the following sites would be best for the screening?

d. The Weight Losers group meeting at the church

Which of the following would be a source of care if families are employed at a salary above the poverty line, but at too low of an income for health insurance?

d. The state Children's Health Insurance Program (CHIP)

Most of a country's citizens have died from malnutrition or as the result of violence. Which of the following conclusions can be drawn?

d. They live in a developing country.

Which of the following offers the best explanation as to why it is important for low-income women to seek prenatal care?

d. To obtain referrals for other helpful services

A 6-month-old infant was brought in unresponsive by both parents. X-ray examination showed evidence of shaken baby syndrome. Which of the following persons is most likely responsible?

dad

Individuals with Disabilities Education Act (IDEA)

ensures a free appropriate public education to children with disabilities, based on their needs in the least restrictive setting from preschool through secondary education

contracting

establishing the division of labor between nurse and family that will meet the objectives

social network framework

explains how social networks can replace the family/friend support network for individuals who feel marginalized in the family system

Civil Rights Act

federal law passed in 1964 that prohibits discrimination based on sex, race, color, religion, or national origin in hiring or firing, wages, and fringe benefits

preconception health

focuses on taking steps in the present to ensure the health of future children, and considering effective contraception if pregnancy is not desired

On which of the following levels of care do most faith community nurses (FCNs) focus?

primary

medical care

the use of ambulatory care, hospital care, preventive care, or other health services

Which of the following statements best explain the difference in suicide rate according to gender?

c. Women are more likely to attempt suicide but less likely to be successful.

anosignosia

lack of awareness, lack of insight in illness

FQHC Look-A-Likes (FQHCLA)

Centers administered by the Health Resources and Services Administration (HRSA), Bureau of Primary Health Care (BPHC), which provide services to underserved populations. FQHCLA sources of funding differ from FQHCs. FQHCLAs do not receive federal grants to fund services, but may receive reimbursement under Medicare and Medicaid (USDHHS, HRSA, BPHC, n.d.).

Federally Qualified Health Centers (FQHC)

Centers administered by the U.S. Health Resources and Services Administration (HRSA), Bureau of Primary Health Care (BPHC). FQHCs receive federal grants to fund services to underserved populations. These centers may receive reimbursement under Medicare and Medicaid (USDHHS, HRSA, BPHC, n.d.).

Which of the following best explains why many elderly women do not prepare meals or do housework?

Chronic diseases, especially arthritis and rheumatism, hinder their ability.

United States Conference of Mayors

This organization provides an annual report on hunger and homelessness in participating major U.S. cities.

Where is youth violence most likely to occur?

inner city schools

cultural competence

the ability to give care to an individual that demonstrates awareness of, and sensitivity to, the underlying personal and cultural reality of the individual by identifying and using cultural norms, values, and communication and time patterns in collecting and interpreting assessment information

illness orientation

the ability to note symptoms and take appropriate action

Which of the following is an accurate definition of impairment? a. A disadvantage resulting from a disability that prevents fulfillment of an expected role b. A restriction or inability to perform an activity in a normal manner c. An anatomical, mental, or psychological loss or abnormality d. A defect that affects society on a macro level

ANS: C A disability, resulting from an impairment, involves a restriction or inability to perform an activity in a normal manner or within the normal range. An anatomical, mental, or psychological loss or abnormality is an impairment. A handicap is a disadvantage resulting from an impairment or disability that prevents fulfillment of an expected role. In a comparison of these concepts, an impairment affects a human organ on a micro level; disability affects a person on an individual level; and a handicap involves society on a macro level of analysis.

On the basis of all the work done by social scientists, approximately how many homeless persons should be expected and tolerated in our society? a. No more than 5% b. No more than 10% c. No more than 15% d. There should not be homeless persons in America.

ANS: D Kozol (1988) argued: "We would be wise to avoid the numbers game. Any search for the 'right number' carries the assumption that we may at last arrived at an acceptable number. There is no acceptable number. Whether the number is 1 million or 4 million, there are too many homeless people in America."

Point-in-Time (PIT) Count

As a requirement of HUD funding, CoCs conduct a Point-In-Time (PIT) count of sheltered homeless people on a single night in late January of every year and submit this data to HUD. Every other year, the PIT Count includes sheltered and unsheltered people who are homeless.

A community health nurse is trying to determine the success of a planned intervention. Which of the following would the community health nurse most likely examine?

Current problem data to compare with original baseline data

individualized education plan (IEP)

a plan outlining the educational program for a child with a disability

psychosocial rehabilitation service (PSR)

a program in which participants learn specific skills that foster rehabilitation and mental health recovery

post-traumatic stress disorder (PTSD)

an anxiety disorder with set characteristic signs and symptoms that occur after a person is exposed to, or is the victim of, a traumatic event

frontier

an area having fewer than six persons per square mile

urban

an area with a high population density

metropolitan

area that contains an urban population center of 50,000 or more people

A parent at a school says to the school nurse, "I can't keep that plate thing straight. What is the bottom line about nutritious eating today?" Which of the following is the most appropriate response by the nurse?

d. "Balance your plate with half of a plate of fruits and vegetables, and a quarter of a plate of both proteins and grains."

A new mother asks the nurse how long she should breast feed. Which of the following statements by the nurse is consistent with the American Academy of Pediatrics (AAP) recommendations for breastfeeding? Chapter 16 pg. 292 slide# 23.

"A mother should combine breastfeeding with the introduction of complementary foods until the infant is 1 year old."

When the school health nurse suggested the school health clinic include sexual counseling and free condoms if requested, an angry woman yelled, "Why do I care if some girls have no morals and sleep around?" Which of the following would be the most appropriate response by the nurse?

"Because taxpayers spend billions each year for teenage moms and their babies."

A nurse asked of a family, "Could each of you tell me what you typically do during the week?" Which of the following would the nurse most likely say to follow up such an open-ended question?

"Could you tell me about what you do at your job?"

Which of the following statements best summarizes Wald and Brewster's approach to home nursing?

"Helping people to help themselves"

adolescent pregnancy

pregnancy before the age of 18

disparities

refers to the wide variations in health services and health status among certain population groups

seasonal workers

those who reside in one place and work when farm labor is needed

Medicaid

Title XIX Social Security Amendment (1965), a combined federal and state program. The program provides access to care for the poor and medically needy of all ages.

A school nurse was responsible for five schools. To give students required medications during the day, the school nurse teaches a particular office staff member in each school exactly what to give to whom and when, and they practice until both are comfortable. Which of the following remains as the nurse's responsibility in relation to the administration of medications?

To systematically continue with assessment, diagnosis, goal setting, and evaluation of student health needs

Which of the following responsibilities does the occupational health nurse (OHN) have in relation to an injured employee and the Workers' Compensation Act?

To work with the employee to keep the employee informed, limit disability, and provide opportunity for rapid return to employment

Which of the following descriptions does NOT reflect a current definition of family? A social unit interacting with the larger society Individuals living together related by kinship, marriage, or choice Mother, father, and children interacting with extended family Two or more persons living together who share common interests

Two or more persons living together who share common interests Current families are diverse in composition and relationships and vary in reasons for living together. Formerly, families were defined by how closely they resembled the "nuclear family" of parents and children, but this is no longer the case. "Family" means any person or persons who play a significant role in an individual's life. DIF: Cognitive level: Knowledge REF: Page 382-383

Social Security

a U.S. federal statute that provides for a national system of old age assistance, survivors' and old age insurance benefits, unemployment insurance and compensation, and other public welfare programs, including Medicare and Medicaid

attention deficit hyperactivity disorder/attention deficit disorder (ADHD/ADD)

a behavior disorder characterized by developmentally inappropriate degrees of inattention, impulsiveness, and hyperactivity

ecological framework

a blend of systems theory and developmental theory, with the addition of an understanding of one's own environment

A nurse is called by her sister, who tells her how wonderful it is that she knows the exact date her baby is coming because her doctor scheduled her for a C-section before his vacation. Which of the following would be the most appropriate response by the nurse?

a. "A C-section is major surgery. Do you remember what the doctor shared were the risks for yourself and your baby?"

In a college health course, the question arose as to when a woman needs to see her health care provider for a Pap smear. Which of the following would be the most appropriate response by the nurse?

a. "After you begin having intercourse or reach age 21, whichever is sooner"

A young mother was very proud because she had been taking her 9-month-old with her to various garage sales where she had purchased a lot of toys for very little money. Which of the following would be the most appropriate response by the nurse?

a. "Be sure there are no small pieces as young children can easily choke."

A nurse is on the treadmill exercising, when the woman next to her says, "You're a nurse. Explain to me how the scale says I've lost weight, but my waist and tummy just keep getting larger." Which of the following would be the most appropriate response by the nurse?

a. "I'm concerned. You need to see your health care provider to follow up on this."

domestic violence

includes physical, sexual, and psychological attacks and economic coercion

Ms. L is being treated for depression. Which of the following statements by her indicates a need for further education? "My medication will cure me of my depression." "A side effect of my medication (Prozac) may be gastrointestinal upset." "I need to keep my follow-up appointment for medication management." "I should not stop taking my medication without checking with my doctor first."

"My medication will cure me of my depression." Psychotherapeutic medications do not cure mental illness; rather, they act by controlling symptoms. The other statements are correct. DIF: Cognitive level: Knowledge REF: Page 481

"I don't understand," a woman said. "I know I'm eligible for Medicare part A and it's free. Part B isn't free. When will I ever use it?" Which of the following would be the most appropriate response by the nurse?

"Part A is only for hospital bills. Most care is given on an outpatient basis. Part B pays a large portion of those costs."

Affordable Care Act (ACA)

health care law offering choices for consumers and provides new ways to hold insurance companies accountable

What was the importance of the Lalonde Report, which proposed the "health field concept" in 1974?

c. It first emphasized influence of lifestyle and environment.

Which of the following best describes why infant mortality rate is used as an international comparison of a country's health status?

c. It reflects medical care quality and access and public health practice.

A client who regularly participates in activities at the local senior center is celebrating his 65th birthday today. Without regard to gender, according to the U.S. average, how many additional birthdays can he anticipate celebrating? 10 14 18 22

18 According to federal statistics, individuals who survive to age 65 years can expect to live an average of nearly 18 more years. DIF: Cognitive level: Knowledge REF: Page 357

Rural residents in the United States compose more than _____% of the nation's poor. 50 35 20 18

50 More than 50% of the nation's poor live in rural areas; 35% live in the South. Current census estimates are that 20% of the nation's children younger than age 18 years live in rural areas, as do 15% of the nation's elderly. DIF: Cognitive level: Knowledge REF: Page 445 50

Children living below the poverty threshold account for what percentage of the U.S. population? 8% 22% 35% 44%

22% Correct In 2013, approximately 22% of American children lived below the poverty line. Many more children (about 44%) live in low-income families that are close to the poverty level and unable to meet basic living expenses. DIF: Cognitive level: Knowledge REF: Page 301

Many barriers exist that prohibit positive male health behavior. One barrier that may exist for men is illness orientation. This is because: A. men may be aware of being ill, but they make a conscious decision not to seek health care to avoid being labeled as "sick." B. men do not have routine reproductive health checkups that include screening. C. men may be less willing to talk, may not recall health problems, and may lack a health vocabulary. D. male health concerns are addressed by specialists and generalists who have not received gender-specific training that would enable them to focus on men's health needs.

A men may be aware of being ill, but they make a conscious decision not to seek health care to avoid being labeled as "sick." Illness orientation is the ability to note symptoms and take appropriate action. However, the stereotypical view of men as strong and invulnerable is incongruent health promotion. Option B is a barrier health protection behavior. Option C is a barrier to reporting health behavior. Option D is a factor related to medical care access that impedes men's health.DIF: Cognitive level: AnalysisREF: Page 344

Public health nurses should be aware that factors that impede men's health include: Select all that apply. A. risk-taking behaviors. B. infrequent use of the health care system. C. gaps in preventative health behaviors. D. lack of male health care providers. E. financial ability.

A B C E

Continuum of Care (CoC)

A CoC is responsible for providing a range of housing and related services, including emergency and preventive responses, to persons in a local area who are experiencing homelessness. A CoC is established through a competitive grant-funded program administered by the U.S. Department of Housing and Urban Development.

Which of the following best describes the incidence of abuse among pregnant women?

Approximately one out of every six women has been abused by a partner.

A nurse assessed carefully and created a comprehensive intervention plan including primary, secondary, and tertiary care for individuals and families. Which of the following factors may the nurse have overlooked?

Are there adequate resources for such a project?

What is one barrier to receiving services that Karen should highlight during her presentation? A. Belief in traditional home remedies B. Easy access to physicians C. Inability to work D. Inability to qualify for low-income programs such as Medicaid

A. Belief in traditional home remedies Belief in home remedies may be a barrier to receiving health services. Personal barriers to health care include culture, language, attitudes, acceptability, education, and income. Members of rural communities usually value the ability to work, and most will not seek medical assistance if they are still able to attend work. Also, they may qualify for low-income programs but choose not to receive these services, even if they are eligible.

The students are aware that many, but not all, of the same health risks apply to women and men. To target those risks related to diseases that cause significantly greater morbidity and mortality in women than in men, which of the following activities would be appropriate? Select all that apply. A. Calcium supplementation education B. Blood pressure screening C. Mammogram education D. Depression screening

A. Calcium supplementation education B. Blood pressure screening C. Mammogram education D. Depression screening Calcium supplementation addresses women's increased risk for osteoporosis and subsequent fractures. Hypertension is more common in women and can be detected with blood pressure screenings. Breast cancer is much more common in the female population, so education regarding recommendations for mammography is appropriate. Depression is more common in women, and screening is an appropriate means to identify those who may be affected.

Assuming Elliston County is similar in demographic trends to the rest of the United States, which of the following data were most likely reflected in the community health assessment? Select all that apply. A. Increasing numbers of children under age 18 B. Increasing numbers of non-Hispanic black children C. Increasing numbers of Hispanic children D. Decreasing numbers of single-parent households

A. Increasing numbers of children under age 18 C. Increasing numbers of Hispanic children There is an increasing proportion of children under age 18, as well as an increasing number of Hispanic children. The number of non-Hispanic black children has remained stable, and there are increasing numbers of single-parent households.

Jada has been an advocate for those with mental illness for many years. In 2000, she was gratified to see major attention given to the issue in the form of the New Freedom Commission on Mental Health. Which of the following would not be a characteristic of the "transformed mental health system" envisioned by the commission? A. Maintenance of current mental health research funding levels B. Elimination of disparity in health insurance coverage for those with mental illness C. Mental health care that is consumer focused D. Mental health care information that is distributed via technology

A. Maintenance of current mental health research funding levels In a transformed mental health system, research funding would be accelerated, not maintained at current (relatively low) levels. All of the other characteristics would be exhibited in a transformed system.

What level of prevention is Kim using when providing education for this group? A. Primary B. Secondary C. Tertiary D. None of the above

A. Primary The primary level of prevention is used when providing education for this group. Primary prevention relates to activities directed at preventing a problem before it occurs by altering susceptibility or reducing exposure for susceptible individuals. Kim is providing education to all employees regardless of whether or not they are at high risk for not seeking medical care

What broad factors may have contributed to the increased incidence of homeless in this community? A. Shortage of affordable housing, insufficient income, and scarcity of supportive services B. Development of a national health care system, insufficient income, and increased section 8 housing C. Stagnation of minimum wage, the opening of new factories, and shortage of affordable housing D. Scarcity of supportive services, war on terror, and increased section 8 housing

A. Shortage of affordable housing, insufficient income, and scarcity of supportive services The three broad factors: (1) shortage of affordable housing, (2) insufficient income, and (3) scarcity of supportive services are societal conditions that contribute to homelessness rather than cause homelessness.

Mr. H, an 85-year-old-man, has recently lost his wife. He comes to the local health department to have his blood pressure checked quite often. The nurse should recognize that Mr. H is at greater risk for what? A. Suicide B. Homicide C. Accidental injury D. HIV infection

A. Suicide Men are five times more likely than women to commit suicide. Men older than age 85 years are 11 times more likely to die as a result of suicide. Men age 15 to 64 years are two to three times more likely to die as a result of unintentional injury than women of the same age. African American men between ages 45 and 64 years are eight times as likely to die of HIV infection as white men. Among men age 15 to 24 years, African American men are seven times more likely to die of homicide.DIF: Cognitive level: AnalysisREF: Page 341

The community health nurse's commitment to health for all requires an increased awareness of men's health issues; however, men are less likely than women to be seen by a community health nurse. This is because: A. maternal and child care are the major focus of many public health departments. B. preventative reproductive health care is the only service available for men. C. community health nurses do not have the competencies to focus on men's health care needs. D. health department hours are flexible.

A. maternal and child care are the major focus of many public health departments. Not only is maternal and child health a major focus of many health departments, but neither a medical nor a nursing specialty within a health department routinely exists to specifically address men's health. Preventive reproductive health care (i.e., family planning, prenatal care, and cancer screening) and associated general screening are not routinely available for men. The hours of services offered by health departments usually do not provide convenient access for men. The community health nurse's commitment to health for all requires an increased awareness of men's health issues in their social and cultural context and individual and group action that will improve men's physical, psychological, and social well-being.DIF: Cognitive level: KnowledgeREF: Page 348

Which of the following best explains why the rural population is growing when young persons from rural areas consistently are moving to urban areas to seek employment? a. Adults engaged in all the physically demanding farming tasks have longer life spans than people in urban areas. b. Farm families continue to have a higher birth rate than urban families. c. Many formerly migrant worker Hispanics are settling in rural counties. d. Retirees and other urbanites able to conduct business through telecommunication and travel are moving in.

ANS: D Recent demographic changes in rural areas have also included an influx of retirees and others from urban areas who are able to live in rural areas and conduct business through telecommunication and travel. The other responses do not support the current reason for the increase in the rural population.

21.) A nurse has recently learned of the death of a 22 year old female friend. Which of the following would be the most likely cause of death? Chapter 17 pg. 317.

Accident or unintentional injury.

Which of the following statements best explains why parents and nurses are often concerned about adolescents? Chapter 16 pg. 298 slide# 29.

Adolescents engage in high-risk behaviors.

Public health policy in the United States is influenced by two types of justice, market justice and social justice. Examples of market justice include which of the following? (Select all that apply.) All people are entitled to the status they create for themselves. All people are entitled to a livable minimum wage. All people are entitled to happiness if they put forth enough effort. Correct All people are entitled to the income they work for. Correct All people are entitled to access to health care.

All people are entitled to the status they create for themselves. All people are entitled to happiness if they put forth enough effort. All people are entitled to the income they work for. Market justice has been the dominant model in the United States and purports that people are entitled to valued ends (i.e., status, income, and happiness) according to their own individual efforts. Moreover, this model stresses individual responsibility, minimal collective action, and freedom from collective obligations other than respect for another person's fundamental rights. In contrast, under a social justice model, all people are equally entitled to key ends (i.e., access to health care and minimum standards of income). DIF: Cognitive level: Knowledge REF: Page 439

18.) In a client's culture it is exceedingly rude to ask any questions about income or wealth. Which of the following actions should be taken by the nurse to draw conclusions about the client's socioeconomic status (SES)? Chapter 13 slide# 18 pg. 228.

Ask about the client's education or employment position.

Which of the following educational topics would be most appropriate in addressing the most common cause of death in women? A. Breast self-examination B. Heart-healthy diet C. Safe sex practices D. Car passenger safety

B. Heart-healthy diet A heart-healthy diet is most important because cardiovascular disease is the most common cause of death in all women. In women younger than 75 years of age, accidents and cancer are the most common causes of death.

Which of the following was required by the Paul Wellstone and Pete Domenici Mental Health Parity and Addiction Equity Act of 2008?

That health insurance must cover treatment for mental illness on the same terms and conditions as physical illness

Karen also would like to address upstream community interventions that local health care providers could use when working with this population. What intervention would she discuss? A. Provide programming about needed health care services B. Interview the church council to learn about meeting the community's health concerns C. Offer medical services at the local health clinic D. Implement a program about lead poisoning

B. Interview the church council to learn about meeting the community's health concerns Interviewing the church council to learn about the community's health needs is an intervention that focuses on thinking upstream. The strategies for upstream interventions include the following: attacking community-based problems at their roots (educate the community about major health issues that affect health and well-being), emphasizing the "doing" aspects of health (encouraging active involvement from the population; looking at aspects of health that involve "doing," such as exercise, diet changes, etc.), and maximizing the use of informal networks (facilitating community involvement in program planning).

Mr. H is a patient in the primary care center where you work as a staff nurse. He tells you that he is currently participating in a stress management program. You know that this is a form of which of the following? Individual therapy Couple therapy Behavioral therapy Family therapy

Behavioral therapy Behavioral therapy uses learning principles to change thought patterns and behaviors systematically; it is used to encourage the individual to learn specific skills to obtain rewards and satisfaction. Stress management, biofeedback, and relaxation training are examples of behavior therapy. Individual therapy focuses on the client's current life and relationships within the family, social, and work environments. Couple therapy is used to develop the relationship and minimize problems through understanding how individual conflicts are expressed in the couple's interactions. Family therapy involves problem-solving sessions with members of a family. DIF: Cognitive level: Knowledge REF: Page 482

Which of the following has been a common result in countries with a severe HIV/AIDS epidemic?

Birth rate is high, but life expectancy is only 35 to 40 years.

4.) Which of the following indicators would be most important when assessing the health of a 1 day old newborn? Chapter 16 slide# 9, pg. 290.

Birth weight and length of gestation.

During the presentation, one of the participants states, "I am turning 65 this year. Who will pay for my visits to the doctor?" How would the nursing student respond? A. "You will be responsible for paying for your doctor visits." B. "Medicare Part A will pay for your doctor visits." C. "Medicare Part B will pay for your doctor visits." D. "You will need a Medigap supplement policy to pay for your doctor visits."

C. "Medicare Part B will pay for your doctor visits." Medicare Part B will pay for your doctor's visits. Medicare Part B covers the costs for physician and nurse practitioner services; outpatient services such as diagnostic procedures, qualified physical, speech, and occupational therapy; ambulance services; durable medical equipment; and some home health services. Medicare Part A is a hospital insurance plan that covers acute care, short-term rehabilitative care, and some costs associated with hospice and home health care.

Tim wants to learn whether Heather is able to access health care for herself and her children. Which question would be the most appropriate for him to ask to learn more about this issue? A. Do you receive annual physical examinations? B. Who takes you to your doctor appointments? C. Are you able to afford the health care services you need? D. Who is your primary care provider?

C. Are you able to afford the health care services you need? "Are you able to pay for the health care services you need?" is the most appropriate question for Tim to ask to learn about Heather's ability to access health care for herself and her children. This question addresses affordability, which is the price of provider services or payment requirements and the client's ability to pay. The other factors that should be addressed when learning about ability to access health care include availability, accessibility, accommodation, and acceptability.

Higher mortality rates for male infants may be explained by which of the following? A. Male infants have a lower risk for prematurity. B. Male infants have a lower rate of respiratory distress syndrome. C. Male infants have a higher rate for infectious diseases. D. Male infants have a higher rate for musculoskeletal disorders.

C. Male infants have a higher rate for infectious diseases. Male infants may a have higher mortality rate because of their higher rates of infectious disease, prematurity, respiratory distress syndrome, and certain types of musculoskeletal disorders.DIF: Reference level: KnowledgeREF: Page 343

Based on current U.S. Census Bureau and American Association of Retired Persons (AARP) information regarding family structure, Connie is reminded that all of the following family structures are increasing in frequency, except which one of the following? A. Single-parent homes B. Grandparent-headed families C. Same-sex couple families D. Cohabiting-couple households

C. Same-sex couple families Although it is possible that the number of same-sex couple families has increased, we have no reliable information about the trend for this family structure because the Census Bureau does not count this information. All of the other family types have a documented increase in frequency of occurrence.

Mr. C, age 40 years, has recently sustained an injury at work. The occupational health nurse knows that based on gender-specific socialization processes, Mr. C would more than likely comply with his medical regimen if he were to: A. remain off work for several weeks. B. retire from his job. C. go back to work in an altered capacity. D. start a new job.

C. go back to work in an altered capacity. Men should be encouraged to return to work in an altered capacity rather than remaining off work, retiring, or starting a new job. Occupational accommodation for treatment regimen will help ensure compliance.DIF: Cognitive level: KnowledgeREF: Page 349

Which of the following characteristics is demonstrated by healthy or energized families? (Select all that apply.) Communication is appropriate and clear. Support and respect are evident and appreciated. Roles are static and change only in times of crisis. Common beliefs and values are shared. Members play and share leisure time appropriately.

Communication is appropriate and clear. Support and respect are evident and appreciated. Common beliefs and values are shared. Members play and share leisure time appropriately. Healthy or energized families communicate, support, teach values, share a religious core, have flexible roles and power, engage in family decision making, foster responsibility and service to family and community, have a sense of humor and play, and grow from problem solving. DIF: Cognitive level: Knowledge REF: Page 389-390

Biogerontology

Connection between aging and disease processes. Bacteria, fungi, viruses are responsible for physiologic changes.

A prisoner was bleeding where another inmate had attacked him with a fork. Which of the following would be the correctional nurse's first and most crucial responsibility?

Maintain an escape route to help ensure personal safety

The first thing that Karen wants to include in her presentation is information about health disparities that exist among this population. What would be the most accurate statement for Karen to include? A. "Rural workers are really just like you and me." B. "Medical access is not a problem for the rural population." C. "It is easy to recruit medical providers to rural communities." D. "Lack of health insurance affects the health services this population receives."

D. "Lack of health insurance affects the health services this population receives." "Lack of health insurance affects the health services this population receives" would be the most accurate statement for Karen to include in her presentation. The lack of health insurance has become a major issue for the health of the nation, especially in the rural population, where seasonal work, low wages, and self-employment all contribute to many families not being able to afford health insurance.

Which of the following is required for an agency to receive reimbursement for care given from Medicare?

Demonstrating meeting federal quality standards for Medicare-covered services

The community health students have been taught that injuries to women are an important concern. Since the health fair will be held on the college campus, where 92% of the students are aged 18 to 25, which of the following educational topics address the injury-related issue of greatest concern? A. Self-defense strategies for rape prevention B. Calcium supplementation C. Car passenger safety D. Crisis intervention for domestic violence

D. Crisis intervention for domestic violence Domestic violence presents the greatest risk for injury in women aged 15 to 44. Rape, injuries from falls, and motor vehicle accidents all present risks for injury, but domestic violence is by far the greatest risk for this age group.

The strategic health planning team has chosen to prioritize the three leading causes of infant mortality. All of the following interventions would be appropriate to address these issues, except which of the following? A. Smoking cessation efforts targeted to pregnant women B. "Back to Sleep" campaign to teach about sudden infant death syndrome (SIDS) prevention C. Folic acid supplementation to prevent certain congenital malformations D. Infant car seat safety information and inspections

D. Infant car seat safety information and inspections The three leading causes of infant mortality are related to chromosomal abnormalities (some decreased risk with folic acid supplementation), low birth weight (increased in pregnant women who smoke), and SIDS (decreased risk in infants sleeping on their back). Infant car seat safety is important, but motor vehicle accidents are not currently a leading cause of infant mortality.

As Connie prepares for the family interview, she is reminded that there are several critical components of the interview. All of the following nursing activities would be consistent with the identified critical components, except which one of the following? A. Properly introducing self and addressing the client and family by name B. Noting and commenting on family members' areas of strength C. Questioning the family about expectations for the meeting D. Pointing out deficits in family functioning

D. Pointing out deficits in family functioning According to Wright and Leahey, all of the above are critical components of the family interview except pointing out deficits in family functioning. It would be appropriate for the interviewer to guide the family in identifying their needs, but it would be inappropriate to unilaterally point out deficits.

ada works with many clients who are affected by mental disorders. She understands that which of the following factors influence mental health? A. Sexual orientation disparities, genetic, and hazardous occupational factors B. Hazardous occupations factors, biological factors, religious factors C. Genetic factors, social factors, and religious factors D. Political factors, genetic factors, sexual orientation disparities

D. Political factors, genetic factors, sexual orientation disparities Treatment of mental disorders has dramatically improved, yet the cause of most mental illnesses is not well understood. Research has identified a number of biological and sociological factors that contribute to mental health and mental illness. Natural and human-made disasters also influence the mental health of individuals, families, and communities, and social and political factors that may be associated with development of mental illness.

suicide

the act or an instance of taking one's own life voluntarily and intentionally

Which of the following strategies should be used to lower the risk of malaria among the world's population?

Distribute insecticide-treated nets and antimalarial medications

A community health nurse needs to understand that most families in today's society can be described as which of the following? Blended Nuclear Extended Defined by its members

Defined by its members The families of today are defined by the members because of the numerous definitions of family. Many different types exist, such as single-parent families, second families or grandparents raising children, gay or lesbian families, or traditional families. DIF: Cognitive level: Knowledge REF: Page 382

What official national organization provides financial housing assistance to low-income families? World Health Organization (WHO) Federation of National Organizations Working with the Homeless (FEANTSA) Robert Wood Johnson Foundation

Department of Housing and Urban Development (HUD) HUD, in cooperation with state and local governments and nonprofit housing organizations, operates programs that provide financial housing assistance to low-income families. The WHO is an organization that is concerned with international public health. FEANTSA is a European organization that works to prevent and alleviate poverty and homelessness in Europe. The Robert Wood Johnson Foundation provides funding for nursing education. DIF: Cognitive level: Knowledge REF: Page 431

A nurse has assessed an aggregate, determined the priority health needs, and decided on what intervention to offer. Which of the following will best determine if the intervention will be successful?

Determination of the aggregate's perspective of the need for the intervention

An elderly client expresses the following concerns to the community health nurse. Which of these concerns is a physiological change associated with the aging process? Increased saliva Decreased pain threshold Increased nightly sleep time Difficulty remembering new names

Difficulty remembering new names With aging, the human body experiences physiological changes in all systems. Included are changes in taste, including diminished taste sensation, decreased saliva production, and decreased sensitivity to sweetness and salt. Pain threshold is increased, perhaps associated with physical changes in brain weight, reductions in functioning neurons, and increases in neurofibrillary tangles. Sleep changes include decrease in stage 3 and 4 sleep, as well as a reduction in total sleep time. Multiple cognitive changes occur as part of normal aging, including better long-term versus short-term memory retrieval. Remembering new names is an aspect of short-term memory. DIF: Cognitive level: Knowledge REF: Page 361

25.) Which of the following best explains how the Civil Rights Act amendments have been helpful to adult women who are employed? Chapter 17 pg. 330.

Discrimination against pregnant women or conditions involving childbirth or pregnancy is prohibited.

For which of the following would a screening test be most appropriate?

Disease b, which can be controlled if caught early in the disease process.

A nurse is developing a community education program about how to prevent sudden infant death syndrome (SIDS). Which of the following information would be included in the presentation? (Select all that apply.) Chapter 16 pg. 293. Slide# 27.

Do not let the baby get too hot during sleep. Avoid exposing the baby to cigarette or cigar smoke. Always place a baby on his or her back to sleep.

A middle-aged woman was obese and being treated for hypertension; and her blood tests showed elevated triglycerides and low high-density lipoprotein (HDL). Besides recommending physical activity, which of the following nutritional guidance should be provided by the nurse?

Eat fish and seafood such as salmon, halibut, tuna, and scallops at least three times a week.

The community health nurse assesses the family as a system within the context of their environments using which approach? Ecological framework Social network framework Transaction model General systems theory

Ecological framework The ecological framework is a blend of systems and developmental theory that focuses on the interaction and interdependence of humans (families) as biological and social beings with the environment. Using this framework the community healh nurse would assess the family as a system within the context of their environments. The social network framework involves all the connections and ties within a group. In the transactional model, the term transaction refers to a system that focuses on process as opposed to a linear approach. General systems theory is a way to explain how the family as a unit interacts with larger units outside the family and with smaller units inside the family. DIF: Cognitive level: Knowledge REF: Page 395

Which of the following is the leading cause of maternal death in the first trimester of pregnancy? Spontaneous abortion Hydatidiform mole pregnancy Hyperemesis gravidarum Ectopic pregnancy

Ectopic pregnancy Ectopic pregnancies are the leading cause of maternal death in the first trimester because of hemorrhage. Spontaneous abortion, hydatidiform mole pregnancy, and hyperemesis gravidarum are not major causes of maternal mortality. DIF: Cognitive level: Knowledge REF: Page 319

Which of the following best describes what the Occupational Safety and Health Act of 1970 requires?

Employers must keep the worksite free from recognized hazards.

Which of the following best explains why companies do not build work places in rural areas where people are anxious to find full-time employment?

Employers need highly educated and skilled employees.

A man has been off work because of a recent illness. Which of the following would be the most useful action for the employer to take to support this man?

Encourage the man to return to work as soon as possible, possibly at something less physically demanding

A nurse was debating about which approach to use to both obtain data about community needs and encourage community participation in their resolution. Which of the following approaches would be most effective in meeting both goals?

Establishing focus groups with selected participants

Which of the following best describes why a nurse working in a U.S. urban area should be knowledgeable about infectious diseases common to areas of Africa?

Every infectious disease is just one airplane ride away.

46.) A school nurse is trying to determine health trends when making plans for health programming in the school for the upcoming school year. Which of the following would be the most appropriate initial action for the nurse to take? Did not find not rationale.

Examine records in the school health office to determine trends.

You work in a community clinic that specializes in pediatric mental health. Joy, age 12 years, has been diagnosed with depression. You know a major risk factor for depression in childhood is which of the following? Family history of diabetes Family history of abuse Family history of depression Family history of poverty

Family history of depression A family history of depression is a major risk factor for childhood depression. Other associated factors that may increase the risk of depression in children and adolescents include a history of verbal, physical, or sexual abuse; frequent separation from or loss of a loved one; poverty; mental retardation; attention-deficit/hyperactivity disorder; hyperactivity; and chronic illness. DIF: Cognitive level: Knowledge REF: Page 477

Which of the following best describes why some advocacy groups have taken a strong stand against physician-assisted suicide?

Fear that some persons may be "encouraged" to accept an early death merely because they are not valued by society for their contribution

Which gender has higher morbidity rates with a higher prevalence of chronic diseases that cause disability and limitation of activities?

Females

A female student is at a fraternity party where alcohol is flowing freely, the music is loud, and people are dancing and talking. Most people seem to be having a great time. What action should the student take, if anything, when she overhears two men talking about having put ketamine in a girl's drink?

First get out of there, then call the campus police and report what was heard

Which of the following statements best describes "thinking upstream"?

Focus on economic, political and environmental factors.

31.) Which of the following is the most useful way for a nurse to discuss aging with older adults? Chapter 19 pg. 356.

Functional age

Which of the following best describes the problem with the viewing disability based on the Nagi model?

Functional limitations are used to determine if an individual is disabled.

Which of the following best describes why all citizens should care about young people joining gangs?

Gangs commit as much as 90% of all crime in some communities.

Which of the following are examples of the internal structure of the family?

Gender and rank order of birth

A male strong firearms advocate stressed his guns were for self-protection and he wanted all criminals to know his family had loaded guns in their home. Which of the following best explains why a nurse would not support his actions?

Guns in the home typically result in dead family members and friends.

35.) A nurse explained that the elderly are twice as likely to die in a home fire as younger persons and therefore suggested some safety measures. Which of the following strategies would the be most important suggestion by the nurse? Chapter 19 pg. 368.

Have smoke detectors installed in the house

A man, who looked about 45 years old, gave a cashier at a grocery store the Special Supplemental Nutrition Program for Women, Infants, and Children (WIC) vouchers for some of the food he had just selected. Which of the following conclusions can be drawn from this scenario?

He had a teenage daughter who was pregnant or with a young child.

43.) Which of the following was suggested by the National Healthcare Disparities Report of 2008 (NHDR) as one reason that there are such disparities in health care outcomes inside the health care system? Chapter 21 pg. 415.

Health care provider bias and poor communication exist.

Which of the following best describes the district nursing service created in the United States by Lillian Wald and Mary Brewster?

House on Henry Street

A public health nurse is working in a rural clinic. On the basis of risk, which of the following would be the priority group for care? a. African American adults b. Adolescents c. Elderly women who live alone d. Migrant and seasonal farmworkers (MSFW)

In general, MSFW may have the poorest health of any aggregate in the United States, and the least access to affordable health care. Eighty-?9?ve percent of the MSFW are Hispanic, Latino, or African American.

Which of the following is a direct result of global warming?

Increased number of parasites and insects

Which of the following best describes why it is important for the nurse to be knowledgeable about the community when providing care to individuals?

Individuals and families are strongly influenced by the community and environment around them.

Environmental Theories

Ingestion of mercury, lead, arsenic, radioactive isotopes, etc. can produce pathological changes. Smoking, air pollutants, crowded living conditions, and high noise levels can have adverse effects on health.

The home care nurse is caring for a client recently diagnosed with inoperable cancer. Which of the following best expresses the nurse's role related to end-of-life issues? Monitoring for the client to express interest in talking about these issues Asking the client whether an advance directive has been completed Exploring end-of-life issues with the client's family Initiating discussion and education

Initiating discussion and education Clients frequently are hesitant to discuss end-of-life issues. This hesitancy arises from a wide variety of psychosocial issues. The nurse-client therapeutic relationship includes proactive communication and education about end-of-life issues, including living will, durable power of attorney or medical surrogate, or do-not-resuscitate order. Discussion regarding end-of-life issues with family members should not be to the exclusion of the client. DIF: Cognitive level: Knowledge REF: Page 374

The composition of the family is an example of which of the following? Internal structure External structure Subsystem Large system

Internal structure Family composition, gender, and rank order are all examples of internal structure. External structure refers to the extended family and larger systems. DIF: Cognitive level: Knowledge REF: Pages 390

Which of the following statements describes the social construct paradigm for disability (as opposed to the medical model)? It focuses on the modification of attitudinal, architectural, sensory, and economic barriers in the environment. It discourages full citizenship for people with disabilities. Health professionals prescribe interventions to remedy functional limitations of the ill person. It focuses on problems and solutions related to the individual.

It focuses on the modification of attitudinal, architectural, sensory, and economic barriers in the environment. This is the only statement that reflects the social construct paradigm, which implies that disability stems from the failure of a structured social environment to adjust to the needs and aspirations of disabled citizens rather than from the inability of a disabled individual to adapt to the demands of society. The other options are characteristics of the medical model. DIF: Cognitive level: Knowledge REF: Page 419

1.) Which of the following best describes why infant mortality rate is used as an international comparison of a country's health status? Chapter 16 slide# 7, pg. 288.

It reflects medical care quality and access and public health practice.

Which of the following best describes why infant mortality rate is used as an international comparison of a country's health status?

It reflects medical care quality and access and public health practice.

Which of the following statements best describes how the Centers for Disease Control and Prevention (CDC) differs from its many peer agencies?

It responds to health emergencies.

A young teenager was habitually engaging in binge drinking with friends on weekends. Which of the following would be a long-term concern of this behavior?

Legal drug use almost always precedes use of illegal drugs.

Which of the following best explains what experts believe is the causative factor in the decline in total alcohol consumption?

Less social tolerance and no-drinking-while-driving campaigns

What is meant by discriminatory land use?

Locating industrial hazards in low-income communities

Higher mortality rates for male infants may be explained by which of the following? Male infants have a lower risk for prematurity. Male infants have a lower rate of respiratory distress syndrome. Male infants have a higher rate for infectious diseases. Male infants have a higher rate for musculoskeletal disorders.

Male infants have a higher rate for infectious diseases. Male infants may a have higher mortality rate because of their higher rates of infectious disease, prematurity, respiratory distress syndrome, and certain types of musculoskeletal disorders. DIF: Reference level: Knowledge REF: Page 343

Which of the following is one of the most challenging areas for the occupational health nurse (OHN)?

Managing ethical conflicts between responsibilities to management and responsibilities to employees

You are working in a mental health facility as a nurse. You are involved with patients who have varied diagnoses. One of your patient's family members asks what biological problem causes all of these people to have mental illness. You respond that most experts believe which of the following? Brain structure is the problem. Neurotransmitters are the problem. Genetics are the problem. Many factors cause the problem.

Many factors cause the problem. Information from studies to date is insufficient to establish a definitive biological cause for mental illness. Scholars have concluded that mental disorders are multifactorial, complex physiological phenomena. DIF: Cognitive level: Knowledge REF: Page 473

40.) Which of the following best describes the current perspective on how communities should treat persons with disabilities (PWDs)? Chapter 21 pg. 412.

Maximize opportunities for PWDs to work and otherwise contribute to community life

Which of the following statements comparing health risks between men and women is true? Men have a higher incidence rate for acute infective and parasitic diseases, digestive system conditions, and respiratory conditions than women. Men are at greater risk for higher morbidity and mortality rates for conditions that are the leading causes of death (e.g., heart disease, cancer, stroke) compared with women. Men are less likely than women to die from unintentional injuries. Men are less likely than women to be involved in criminal activities.

Men are at greater risk for higher morbidity and mortality rates for conditions that are the leading causes of death (e.g., heart disease, cancer, stroke) compared with women. Men have higher morbidity and mortality rates for conditions that are the leading causes of death. Women have a higher incidence of acute infective and parasitic diseases, digestive conditions, and respiratory conditions than men. Men are more likely to die from unintentional injuries and to be involved in criminal activities. DIF: Cognitive level: Knowledge REF: Page 341

Which of the following statements best explains why men are sicker on hospital admission than women?

Men do not seek treatment until they are very ill.

Which of the following best explains why some community health nurses find nursing models inappropriate for use in their clinical practice?

Models, developed for the middle class, do not fit low-income families.

Mental illnesses adversely affect achievement of social justice. Which of the following statements related to marginalization of some populations because of mental illness is FALSE? About half of all inmates in jails and prisons have mental health problems. Mental illnesses can lead to employment difficulties, which adversely affects access to health insurance, thereby limiting treatment options. Most mental illnesses are caused by genetic factors; therefore, problems associated with them persist across generations and among certain racial and ethnic groups. Both natural and human-made disasters can adversely affect mental health; those with limited resources appear to be more directly impacted, and the effects persist longer.

Most mental illnesses are caused by genetic factors; therefore, problems associated with them persist across generations and among certain racial and ethnic groups. Although genetic factors most likely contribute to some mental illnesses and in some individuals there is little information linking a specific gene to a specific disorder. Rather, the major psychiatric disorders are complex, and mental illnesses appear to result from a very complex interaction among neurochemical and metabolic changes and environmental and situational factors. The other statements are all true. DIF: Cognitive level: Knowledge REF: Page 473

According to the ICF (International Classification of Functioning, Disability, and Health), which of the following would be classified as a disability? Detached retina Need to use a powered wheelchair Spinal cord injury Amputation

Need to use a powered wheelchair A disability involves any restriction or lack of ability (resulting from an impairment) to perform an activity in a normal manner or within the normal range. Using a powered wheelchair is the only option that fits this description. The other options would be classified as impairments. DIF: Cognitive level: Analysis REF: Page 405

3.) As a community health nurse, you are concerned about the variable that can affect infant mortality rates. Select the factors that can increase infant mortality rates. (Select all that apply). Chapter 16 slide# 6,7,9...possible "c" as an answer. Slide# 17? Pg. 299?

No prenatal care. Educational level less than a high school diploma. **** Nonsmoker during pregnancy Smoker during pregnancy.

6.) A nurse is caring for a woman, who has just found out she is pregnant. The woman begins crying and says, "But I'm not trying to get pregnant!" Should this response by the woman be surprising to the nurse? Chapter 16 slide# 18, pg. 290.

No, because about half of all pregnancies in the United States are unintended.

19.) A woman, from a culture different from the nurse, stood very close to the nurse and asked a question about her husband's condition. The nurse took a step back and shared the requested information. No other questions were asked. Was this a successful interaction? Chapter 13 pg 232.

No, because the woman and the nurse had different perceptions of personal space.

Which of the following best describes the mission of the U.S. Department of Homeland Security (DHS)?

To protect against and respond to threats and hazards to the nation

An emergency department nurse was not sure whether there was enough evidence one way or the other to suspect child abuse with a child client, because there was not much evidence of physical harm. The forensic nurse agreed to see the child. Which of the following would be the forensic nurse's primary focus?

Observe the child's appearance and behavior and how the child interacted with other children and adults

What is the purpose of the Homeless Information Management System (HIMS)? Provides means to feed homeless people Obtains national data on homeless individuals Serves as a screening tool to obtain any history of criminal activity Provides homeless individuals with educational materials

Obtains national data on homeless individuals The intent of the HIMS is to obtain national data, including the number of and demographic information on the homeless population. Providing means to feed homeless people, serving as a screening tool to obtain any history of criminal activity, and providing homeless individuals with educational materials are not functions of the HIMS. DIF: Cognitive level: Knowledge REF: Page 432

23.) On each home visit, the nurse reviews the medications taken by her elderly clients. Which of the following best explains the rationale for the nurse's action? (Select all that apply.) Chapter 19 pg. 364.

Older adults frequently use many over-the-counter medications, as well as folk or herbal remedies. Age-related changes and poly-pharmacy make elderly clients vulnerable to drug interactions and dangerous adverse reactions. With the high prevalence of chronic diseases, elderly clients are taking a large number of medications.

45.) A family has recently learned that their child will be permanently disabled. The parents do not deny the disability, but do not fully realize its impact. Which of the following levels of adjustment are the parents experiencing? Chapter 21 pg. 417.

Ostrich phase

Wear and Tear Theory

Over time cells wear out due to use & lack of ability to rejuvenate themselves. Aggravating factors include toxins, radiation, uv light, stress, smoking, malnutrition, injury to tissue, and excessive alcohol.

Error Thory

Over time, errors (mutation) occur in the transcription of the steps of protein synthesis of RNA & DNA.

How does participatory action research differ from other research methods?

Participatory action research poses solutions to the problem.

One hundred women received notification that their screening tests suggested that they might have a serious health problem. Which of the following actions should the nurse take to ensure that they will complete follow-up testing?

Point out that screening is only suggestive and that not all positive screenings mean they actually have the health problem.

Which of the following best explains why cancer rates increase as people get older?

Poor lifestyle choices, environmental exposures, and increased life expectancy have all caused cancer rates to increase.

What is the greatest threat to child health? Accidents Racism Burns Poverty

Poverty. Correct Poverty is the greatest threat to child health. Child poverty in the United States is higher than in most other industrialized countries and the rate is rising. Low socioeconomic status contributes to most factors associated with childhood mortality. DIF: Cognitive level: Knowledge REF: Page 301

A community health nurse is reviewing the results of SIPP (Survey of Income and Program Participation) used by the U.S. Census Bureau to collect disability data. Which of the following will the nurse find classified as an instrumental activity of daily living? Seeing Walking Dressing Preparing meals

Preparing meals Instrumental activities of daily living involve such things as going outside the home, shopping, tracking money or bills, light housecleaning, and preparing meals. Seeing and walking are functional activities. Dressing is an activity of daily living. DIF: Cognitive level: Knowledge REF: Page 410

A nurse wants to make sure that the teaching materials and the communication channels being planned for use during an educational program will be effective. Which of the following actions should the nurse take?

Pretest the materials with members of the target group to obtain feedback

A forensic nurse is responsible for conducting a thorough examination, including a history, performing a physical assessment, and collecting forensic evidence on a female client following a rape. In which of the following subspecialties of forensic nursing is this nurse practicing?

Sexual assault nurse examiner (SANE)

Which action would probably result in the largest change in health care outcomes for Americans?

Redirect a large portion of federal funding from acute care to health promotion activities

Tony, age 16 years, comes to see you, the school nurse, with complaints of abdominal pain. You note that Tony exhibits signs and symptoms of depression, states he wishes he did not wake up today, and exhibits signs of anxiety. You would do which of the following? Refer him immediately for evaluation. See him back in your office next week. Refer him next month for evaluation. Do nothing; this is normal in a 16-year-old boy.

Refer him immediately for evaluation. It should be noted that suicide is the third leading cause of death among persons 15 to 25 years of age and that the rate of suicide among young men is five times that among young women. Thus, it is important that all community health nurses become familiar with assessing for suicide warning signs and accessing appropriate resources. Whenever individuals exhibit suicide warning signs, nurses should refer the person to a mental health clinic or provider as soon as possible. DIF: Cognitive level: Analysis REF: Page 480

The nurse knows that cultural competence is defined as: Chapter 13 slide# 2 pg.221.

Respecting and understanding the belief and values of a particular cultural group.

A school nurse performs all of the following activities. Which one demonstrates primary prevention? Establishing an after-school group for latch-key children Reviewing immunization records Obtaining grant funding to test for serum lead levels Vision and hearing screening

Reviewing immunization records. Correct The only option above that represents primary prevention is reviewing immunization records. This is a teachable opportunity for the nurse to stress the importance of maintaining immunizations for the child. The other options are secondary or tertiary prevention activities. DIF: Cognitive level: Application REF: Page 294

A community health nurse is overwhelmed with all that needs to be done in one day. Which task could most easily be postponed?

Reviewing the most recent hospital patient data collected by the local college of nursing

Mr. T has been admitted to the mental health floor for evaluation. You, the nurse, note that he has displayed hallucinations, disorganized thinking and speech, and bizarre behavior. He has a flat affect, lack of energy, and poor attention. You know that these are symptoms of what disorder? Schizophrenia Depression Anxiety Panic disorder

Schizophrenia Diagnostic criteria for schizophrenia are two or more of the following (each present for a significant portion of a 1-month period): delusions, hallucinations, disorganized speech (e.g., frequent derailment or incoherence), grossly disorganized or catatonic behavior, or negative symptoms (e.g., affective flattening, alogia, avolition). DIF: Cognitive level: Application REF: Page 476

Which of the following best describes how Lillian Wald helped encourage the idea of school nurses in the schools?

She demonstrated that school nurses decreased absenteeism by 50%.

Affordable Care Act (ACA)

Signed into law by President Barack Obama on March 2010; provides Americans with better health security by putting in place comprehensive health insurance reforms

Which of the following health-related variables is examined by the critical theoretical perspective?

Social inequalities

Which model supports upstream thinking with the purpose to improve homelessness through reduction of structural conditions contributing to homelessness? Social justice Market justice Physical justice Mental justice

Social justice The social justice model seeks to reduce the structural conditions contributing to homelessness through collective action, thus supporting upstream thinking. DIF: Cognitive level: Knowledge REF: Pages 439-440

A nurse had all the details carefully arranged for a project: location, speaker, seating arrangement, refreshments, handouts, visual aids, and students to distribute and collect evaluation sheets. Which of the following aspects of the project may be a problem?

Something totally unexpected and unplanned for happened.

Which of the following helps to balance the fact that poverty is typically a major factor in certain groups having poorer health?

Strong family and community support for healthy behaviors

The daily patrons of a local town restaurant became ill with nausea, vomiting, and diarrhea. Without knowing the cause, which of the following actions should be taken immediately to help avoid future problems?

Strongly emphasize the importance of proper hand washing to staff

The community health nurse is aware that one of the five leading causes of infant death in the United States is: respiratory distress. circulatory system disease. neonatal infections. sudden infant death syndrome (SIDS).

Sudden infant death syndrome (SIDS) The five leading causes of infant death are congenital defects, disorders related to short gestation or low birth weight, sudden infant death syndrome (SIDS), maternal complications of pregnancy, and accidents such as suffocation. These five factors account for close to 60% of all infant deaths. Respiratory distress of the neonate, diseases of the circulatory system, and neonatal sepsis are in the top 10 leading causes of neonatal mortality in the United States. DIF: Cognitive level: Knowledge REF: Page 288

A young woman is panic-stricken. She had not realized how much she was drinking, and she thinks she had unprotected sex with several men last night. She is petrified and wants an HIV test immediately. Which of the following actions should be taken by the nurse?

Suggest she be treated immediately to be safe

A nurse was responsible for setting up a health fair with free pizza being served during lunch. The nurse observed a few people who did not eat but just talked quietly during lunch. Which of the following actions should be taken by the nurse?

Talk to the group, and ask what foods might have been more acceptable

A nurse is using the theory of reasoned action when working with a client to exercise on a regular basis. Which of the following would be the easiest way for the nurse to determine if the client will engage in this activity?

The clients intention to exercise regularly

The male home health nurse parked his marked car in preparation for visiting the next client when a young female walked over to the car and asked him if he would like to have some fun. Which of the following provides the best explanation for this behavior?

The girl may have hoped a nurse would give her enough money to eat.

If not at a meeting or giving an educational program, which of the following best describes why an occupational health nurse (OHN) would be absent from the health station during the nurse's employment hours?

The nurse planned a walk-through of the workplace.

Which of these statements is true regarding income insufficiency in the United States? The poverty rate increased between 2007 and 2010. People who make minimum wage would not be categorized as impoverished. Alcoholism is the major reason for lack of employment. The median household income in 2011 was around $40,000.

The poverty rate increased between 2007 and 2010. The poverty rate increased from 2007 to 2010. The median income in 2011 was greater than $50,000. The economy and related issues are the most common reasons for unemployment. Working at minimum wage places a person below the poverty line. DIF: Cognitive level: Knowledge REF: Page 435

Which of the following best describes what is happening with infectious diseases in the United States?

The rate of infection varies extensively by income and ethnic groups.

Which of the following best explains the purpose of the Outcome and Assessment Information Set (OASIS)?

To evaluate and improve clinical performance quality

30.) Which of the following best explains why all family members should want their aged relatives to have a living will? Chapter 19 pg. 374..

To help family members know what their loved one does or does not want if he or she becomes unable to speak for themselves.

Which of the following is the primary factor in how a hospice nurse makes decisions while giving care in the client's home, surrounded by the client's family?

To prevent a decline in the caregiver's health

34.) A nurse needed to do a quick assessment of a client's lifestyle behaviors to counsel appropriately. Which of the following strategies would be the easiest way to determine the elderly woman's nutritional status? Chapter 19 pg. 364

Use the DETERMINE mnemonic

A nurse is completing a health history with a client. Which of the following questions would be most important to include?

What kinds of tobacco do you use daily?

Which of the following best describes the time period when communities began to agree on collective action to stay healthy?

When people were nomads engaged in hunting and gathering

When would the National Guard become involved in a disaster?

When the state has been asked to help an overwhelmed community

fetal alcohol spectrum disorder (FASD) a

nervous system dysfunction in infants caused by high consumption of alcohol by pregnant women

A community health nurse may work in a collaborative role with which program that provides nutritious foods, nutrition education, and referral to millions of American pregnant and nursing women and infants and small children? Medicaid School-based health services Women, Infants, and Children (WIC) Maternal and Child Health (MCH) Block

Women, Infants, and Children (WIC) Correct WIC is a federally funded program that provides highly nutritious foods, nutrition education and counseling, and referral to millions of low-income pregnant and breastfeeding mothers and their infants and small children. Medicaid is a health insurance program for poor and low-income people. School-based health centers typically provide a combination of screening and preventive services, primary care, mental health and substance abuse counseling, dental health, nutrition education, and other health promotion activities. The MCH Block Grant program allocates federal funds to the states, and the states must contribute their own funds for maternal and child health services. DIF: Cognitive level: Knowledge REF: Page 306

After completing a master's degree, a nurse took a course in marketing. Should the agency reimburse the nurse's tuition costs?

Yes, the nurse could use such information in social marketing for the agency

Which of the following strategies should be used to lower the risk of malaria among the world's population?

a. Distribute insecticide-treated nets and antimalarial medications

Which of the following best describes the screening tests that are completed on all newborns in the United States?

a. It varies from state to state, somewhere between 0 and 25 tests.

In what way is the U.S. approach to primary health care inconsistent with the World Health Organization (WHO) approach?

a. Our primary health care system does not provide universal coverage to all citizens.

The nurse is aware that a risk factor for elder abuse is: a caregiver with low income. an elder with urinary incontinence. Correct an elder who is irritable and demanding. a caregiver with a strong need for control.

an elder with urinary incontinence. Individuals who are most at risk for elder abuse include female widows age 75 years and older, elders dependent on a caregiver for food and shelter, those with incontinence, and individuals who are frail or have illness or mental disability. Studies have described the typical abuser as a family member (typically a son or daughter) who is middle age or older with low self-esteem and low impulse control. The caregiver's perception of his or her own stress is strongly related to the risk for abuse. Elder abuse occurs at all cultural and socioeconomic levels. DIF: Cognitive level: Knowledge REF: Page 369

A mother explained that she had just dropped her 3-year-old off at her own mother's so she could come in for the required examination for her new job. She excitedly shared her hopes that she would be able to afford a used car rather than having to ride a bus all the time. Which of the following would be the most appropriate response by the nurse?

b. "Don't forget to buy a child's seat when you get your car."

"I don't want to listen to another lecture on safe sex. Just give me my antibiotic and let me leave," said a defiant client. Which of the following would be the best response by the nurse?

b. "Gonorrhea is becoming resistant to antibiotics. It's becoming difficult to treat you, and permanent damage to your body is occurring."

Which of the following statements best define epidemiological transition?

b. A change from mainly infectious diseases to chronic diseases

A nurse has recently learned of the death of a 22-year-old female friend. Which of the following would be the most likely cause of death?

b. Accident or unintentional injury

Which of the following is the drug of choice for treating persons with alcohol withdrawal?

benzodiazepines

A school nurse is trying to determine health trends when making plans for health programming in the school for the upcoming school year. Which of the following would be the most appropriate action for the nurse to take?

b. Do an online search of groups that might collect and analyze such data

A nurse is caring for a woman, who has just found out she is pregnant. The woman begins crying and says, "But I'm not trying to get pregnant!" Should this response by the woman be concerning to the nurse?

b. No, because about half of all pregnancies in the United States are unintended

A 50-year-old male client should be counseled to receive the following preventive health screenings: (Select all that apply.) blood pressure check every 2 years. dental examination every year. eye examination every 3 to 5 years. colorectal screening every 3 to 5 years. abdominal aortic aneurysm screening every 2 to 3 years.

blood pressure check every 2 years. dental examination every year. eye examination every 3 to 5 years. colorectal screening every 3 to 5 years. One-time abdominal aortic aneurysm is recommended for all men older than age 65 years who have smoked. Blood pressure check every 2 years, dental examination every year, eye examination every 3 to 5 years, and colorectal screening every 3 to 5 years are recommended preventive health screenings for men older than the age of 50 years. DIF: Cognitive level: Knowledge REF: Page 349

A woman confides in the nurse that she and her husband want to become parents as soon as possible. Which of the following statements would be the best reply from the nurse?

d. "Start living as healthy a lifestyle as possible right now."

A mother was debating whether or not to take her son to the clinic. His temperature was 104° F and rising so she gave him acetaminophen (Tylenol). Which of the following will most likely be the deciding factor in the mother's decision?

d. Whether or not the child is covered under insurance

cardiovascular disease

includes high blood pressure, coronary heart disease, stroke, congenital defects, and rheumatic heart disease

Family and Medical Leave Act (FMLA)

enacted in 1993, allows an employee a minimum provision of 12 weeks of unpaid leave each year for family and medical reasons such as personal illness, an ill child, parent, or spouse, or the birth or adoption of a child

Health prevention activities targeting the 13- to 18-year-old population must be based on the knowledge that a priority health threat faced by adolescents is: inadequate problem solving and coping skills. avoidance of preventive health services. engaging in risk-taking behaviors. sexual activity.

engaging in risk-taking behaviors. Correct The major developmental goal of adolescence is establishing independence. When coupled with adolescents' view of themselves as invincible, this often leads to risk-taking behaviors. Although early and unprotected sexual activity is one example of risk-taking behavior, the best (more comprehensive) answer is engaging in risk-taking behaviors. Generally, adolescence is a time of emotional maturity that includes learning problem-solving and coping skills. Adolescents are generally healthy, and it is common that they rarely use preventive health services. DIF: Cognitive level: Analysis REF: Page 298

Family interviewing requires the community health nurse to understand or develop specific skills. All of the following demonstrate appropriate family interviewing skills or techniques except: introducing oneself by name and addressing the client appropriately, such as "Mr. Jones." planning the focus for the family visit in advance and engaging all family members present during the home visit. establishing the nursing goals based exclusively on the physician's orders. including time to identify family strengths and commend the family on these during the family assessment.

establishing the nursing goals based exclusively on the physician's orders. Family interviewing skills include social skills (manners), therapeutic conversations, use of tools such as the genogram or ecomap, use of therapeutic questions, and commending family and individual strengths. Assessment, goals, and interventions are completed and planned in collaboration with the family. DIF: Cognitive level: Knowledge REF: Pages 385-386

Mr. C, age 40 years, has recently sustained an injury at work. The occupational health nurse knows that based on gender-specific socialization processes, Mr. C would more than likely comply with his medical regimen if he were to: remain off work for several weeks. retire from his job. go back to work in an altered capacity. start a new job.

go back to work in an altered capacity. Men should be encouraged to return to work in an altered capacity rather than remaining off work, retiring, or starting a new job. Occupational accommodation for treatment regimen will help ensure compliance. DIF: Cognitive level: Knowledge REF: Page 349

Based on theory, which of the following are men most likely to do when they become ill?

ignore symptoms

morbidity

illness rate; the rate at which an illness occurs in a particular population

The community health nurse is reviewing the Healthy People 2020 objectives. She understands a primary goal for the elderly population is: decrease the proportion of older adults who have diabetes. increase the proportion of older adults with reduced physical or cognitive function who engage in light, moderate, or vigorous leisure-time physical activities. increase the proportion of older adults who qualify for Medicare benefits. increase the proportion of older adults who seek medical care for chronic conditions.

increase the proportion of older adults with reduced physical or cognitive function who engage in light, moderate, or vigorous leisure-time physical activities. Healthy People 2020 establishes national objectives for health promotion and disease prevention. Healthy People 2020 has incorporated specific objectives related to older adults that are designed to promote healthy outcomes for this population, including increasing the proportion of older adults with reduced physical or cognitive function who engage in light, moderate, or vigorous leisure-time physical activities; increasing the proportion of older adults who receive Diabetes Self-Management Benefits; increasing the proportion of older adults who use the Welcome to Medicare benefit; and increasing the proportion of older adults with one or more chronic health conditions who report confidence in managing their conditions. DIF: Cognitive level: Knowledge REF: Page 360

Which of the following facts is noteworthy about adolescent males who are driving without a license in the countryside?

most were hispanic or black from rural areas

In long-range planning for community needs, the community health nurse considers that the fastest-growing segment of the senior population is the: young-old (ages 65-74 years). middle-old (ages 75-84 years). old-old (ages 85 years and older). Correct elite old (more than 100 years old).

old-old (ages 85 years and older). The greatest growth will occur in the population aged 85 years and older, whose numbers are projected to grow from 5.5 million in 2010 to nearly 19 million by 2050. DIF: Cognitive level: Knowledge REF: Page 357

schizophrenia

presents with (1) positive symptoms including hallucinations, delusions, disorganized thinking and speech, and bizarre behavior and (2) negative symptoms, such as flat affect, poor attention, lack of motivation, apathy, lack of pleasure, and lack of energy. Onset typically occurs during late adolescence and early adulthood in males and somewhat later in females. There is an increased risk for alcohol use, depression, suicide, and diabetes among persons with schizophrenia.

The nurse could not find a vaccine in the refrigerator. The unopened vaccine vial had been put in a storage cabinet with the other medications. The nurse drew up the appropriate dose in the syringe and then put the vial of vaccine in the refrigerator. Which of the following would be the most likely result?

primary vaccine failure

deinstitutionalization

process of releasing patients from care in psychiatric institutions into the community

Education for Homeless Children and Youth (ECHY) The Education for Homeless Children and Youth (ECHY)

program provides services to homeless children and youth who are not included in HUD's programmatic services. ECHY is administered by the U.S. Department of Education.

Special Supplemental Nutrition Program for Women, Infants, and Children (WIC)

provides highly nutritious foods, nutrition education and counseling, and screening and referral to needed services for low-income pregnant and breastfeeding mothers, their infants, and their children under 5 years of age

infant mortality

rate of death among infants during the first year of life; an important gauge of overall children's health status and the health of pregnant women

family planning

refers to planned limitation of pregnancies or attempts to increase chances of conception

Public health nurses should be aware that factors that impede men's health include: (Select all that apply.) risk-taking behaviors. Correct infrequent use of the health care system. Correct gaps in preventative health behaviors. Correct lack of male health care providers. financial ability. Correct

risk-taking behaviors. Correct Although there are many male health care providers, there are not many health care providers that specialize in men's health. Little effort has been made to create a male-specific health care climate. Risk-taking behaviors, infrequent use of the health care system, gaps in preventative health behaviors, and financial ability are all factors that impede men's health. DIF: Cognitive level: Knowledge REF: Page 345

Which of the following actions should be taken during a biological attack?

seek medical attention if you become sick

Migrant and seasonal farm workers constitute a high-risk population due to their low income and migratory status. These workers lack adequate access to preventive services. The most important role of the rural community health nurse in meeting the health care needs of this vulnerable population is to: address the multiple communicable diseases of these farm workers. serve as an advocate working to gain health care access for these farm workers. participate in political activities with an emphasis on changing immigration laws. learn to speak Spanish to improve communication with this specific population.

serve as an advocate working to gain health care access for these farm workers. One of the greatest needs of the migrant and seasonal farm workers is the lack of access to health care. The community health nurse is in a strategic position to advocate for change to restructure health services and thus reduce this rural health disparity. Providing direct care for communicable diseases is important but does not meet the overarching goal of getting adequate health care access to migrant farm workers. Participating in political activities with an emphasis on changing immigration laws and learning to speak Spanish to improve communication with this specific population do not address the healthcare needs of migrant farm workers. DIF: Cognitive level: Application REF: Page 457

case management (CM)

service to assist clients with serious mental illness and or substance abuse problems in areas of, but not limited to, housing, finances, social, family, medical, medication management, transportation

psychotropic/psychotherapeutic medications

sometimes called psychoactive drugs; affect the central nervous system and can cause a variety of changes in behavior or perception

American Cancer Society (ACS) (2012) recommendations for breast self-exam (BSE) include: (Select all that apply.) BSE should be taught to women beginning their 40s. the correct technique for BSE should be taught to improve detection of abnormalities. BSE should be performed monthly. women at higher risk of breast cancer may benefit from regular BSE. it is acceptable for women to choose not to perform BSE.

the correct technique for BSE should be taught to improve detection of abnormalities. women at higher risk of breast cancer may benefit from regular BSE. Correct it is acceptable for women to choose not to perform BSE. Correct Studies support that BSE contributes to awareness, helping women be alert to changes in their breasts. The ACS (2012) recommendations for BSE are: · BSE benefits and limitations should be taught to women beginning their 20s. · The correct technique for BSE should be taught to improve detection of abnormalities. · Recommendations for the frequency of BSE are no longer specified. · It is acceptable for women to choose to perform BSE, and this practice should not be discouraged. · It is acceptable for women to choose not to perform BSE. · There may be an added benefit for the woman at higher risk of breast cancer to perform regular BSE. DIF: Cognitive level: Knowledge REF: Page 324

A nurse overheard an adult woman, shaking her head in exasperation as she said, "Dad, you're being unreasonable. It's not safe!" Which of the following is the most likely reason for this argument?

the dad wants to keep driving

maternal mortality

the death of a woman while pregnant or within 42 to 364 days after termination of pregnancy, irrespective of the duration and the site of the pregnancy, from any cause related to or aggravated by the pregnancy or its management, but not from accidental or incidental causes

The greatest promise for improving health outcomes for rural residents is: the increase in health care providers migrating to rural areas. the expansion in Medicare services to rural hospitals and clinics. the increase in distribution of technology, providing more education and health services. the expansion of public transportation, allowing rural residents to travel to tertiary care hospitals for specialty services.

the increase in distribution of technology, providing more education and health services. Rural patients are able to access specialty services such and radiologic or dermatologic examinations through telemedicine, and rural people are increasingly taking advantage of the Internet to access information to make health decisions. Expansion and increased distribution of technology in rural areas has increased access to health care services and health education through distance programs for rural residents. Health care providers remain scarce in rural areas. Rural hospitals and clinics generally have Medicare services. There is still inadequate public transportation from most rural areas to tertiary hospitals. DIF: Cognitive level: Analysis REF: Page 463

Which of the following definitions of family would be the most useful for the nurse practicing in the community?

whoever the family says is in their family

A mother explained that she had just dropped her 3-year-old off at her own mother's so she could come in for the required examination for her new job. She excitedly shared her hopes that she would be able to afford a used car rather than having to ride a bus all the time. Which of the following would be the most appropriate response by the nurse?

"Don't forget to buy a child's seat when you get your car."

A principal called the school nurse into his office and asked, "Will this student be able to attend our school?" after handing over the medical record of a severely handicapped prospective student. Which of the following would be the most appropriate response by the nurse?

"Legally all students have a right to public education in the least restrictive environment possible, so let me think how we can manage."

During a health history, the client looked confused and said, "But everybody drinks, like at parties and such. How much drinking do you think is too much?" Which of the following would be most appropriate response by the nurse?

"Per day, no more than one drink for women and two drinks for men."

Military Sexual Trauma (MST)

"Sexual assault or repeated, threatening sexual harassment that occurred while the veteran was in the military. It includes any sexual activity where someone is involved against his or her will" (U.S. Department of Veterans Affairs, 2012).

7.) A woman confides in the nurse that she and her husband want to become parents as soon as possible. Which of the following statements would be the best reply from the nurse? Chapter 16 slide# 18, pg. 290.

"Start living as healthy a lifestyle as possible right now."

36.) A son explained to the home health nurse that since his mother died, his father did not seem to have any interest in life and had even told his son he was considering suicide. Which of the following would be the most appropriate response by the nurse? Chapter 19 pg. 372 slide#19.

"Suicide is a serious concern; let's talk to his doctor about what you've noticed."

A male nurse is enjoying a very hot summer day at the zoo when he notices a family approaching. An elderly member of the family is moving very slowly and being scolded by another adult, saying "You are not sick. Let's keep moving." The nurse touches the elderly adult who has a flushed face and notices clammy and moist skin. What should the nurse say to the adult?

"Take your family member to the air-conditioned restaurant, and let your family rest for a while."

9.) A mother was very proud that she had cut back smoking from two packs a day, which she had smoked for years, to only half a pack a day. Which of the following would be the most appropriate response by the nurse? Chapter 16 pg. 291, 310 (interventions) slide# 20.

"That must have been difficult, but your baby is still getting nicotine, so it's important you quit entirely."

I want to get pregnant as soon as possible now that I'm married," a 43-year-old female says to the nurse. Which of the following would be the most appropriate response by the nurse? Chapter 17 pg. 327.

"That's great, but you need to know that there are more risks for you at your age."

"I want to get pregnant as soon as possible now that I'm married," a 43-year-old female says to the nurse. Which of the following would be the most appropriate response by the nurse?

"We will help you, but you need to know that there are more risks for you at your age."

A young man rolled his wheelchair up the ramp into the clinic to the desk where the intake nurse greeted him. After welcoming him to the clinic, which of the following should be the initial question asked by the nurse?

"What brings you to the clinic today?"

41.) The mother confided in the nurse, "I don't think my child is improving." Which of the following statements would be the best response by the nurse? DID NOT FIND RATIONALE.

"What is causing your concern?"

A man had been in an extended-care facility for five weeks and was now home. His wife tells the nurse, "He's trying to take up where he left off but we've adapted to not having him home. He keeps thinking things will be just as they were when he left, but they aren't." Which of the following would be the most appropriate response by the nurse?

"You all adapted to running the house without him; it will be challenging now to adapt back."

32.) A man stated, "I used to work out at the exercise room at work, but now that I'm retired I sit and watch TV. My wife keeps nagging me to do something. What do you think I should be doing at my age?" Which of the following would be the most appropriate response by the nurse? Chapter 19 pg. 363.

"You should continue to exercise. Is there a gym or health club nearby?

cohabitation

"a living arrangement in which an unmarried couple live together in a long-term relationship that resembles a marriage," (U.S. Census Bureau, 2008, 2010).

Which of the following females would be at greatest risk for developing bulimia nervosa?

15 year old cheerleader who wants to be a ballerina

In which era were people with mental disabilities labeled as "feebleminded" because they could not reach the minimum educational level? Biblical times 18th century 19th century 20th century

19th century During the 19th century, the Industrial Revolution stimulated a societal need for increased education. Those who could not reach the minimum education level were labeled as "feebleminded." In Biblical times, people with disabilities were often viewed as unclean or sinful. In the 18th and 19th centuries, people saw disability as an irreparable condition caused by supernatural agency. Special interest groups for people with disabilities began to develop in the 20th century. DIF: Cognitive level: Knowledge REF: Page 409 19th century

Physiological gender differences that may account for increased mortality rates for men include all of the following except: Select all that apply. A. male immune systems are weaker than women's. B. men have higher levels of stored iron than women. C. men have fat stores in their abdomen. D. men's testosterone levels are cardiac protective. E. men's brain cells die faster than women's.

A B C E

A 50-year-old male client should be counseled to receive the following preventive health screenings: Select all that apply. A. blood pressure check every 2 years. B. dental examination every year. C. eye examination every 3 to 5 years. D. colorectal screening every 3 to 5 years. E. abdominal aortic aneurysm screening every 2 to 3 years.

A B C D

Which of the following would be considered a long-term disabling condition in which the person must learn to incorporate the modifications required for living into daily living and identity? A sprained ankle requiring the use of crutches A total knee replacement surgery requiring the use of a walker after surgery A car accident with a broken arm requiring surgery A cardiovascular accident (CVA) with paralysis to the right side of the body

A cardiovascular accident (CVA) with paralysis to the right side of the body Those with a temporary disability have a very different experience from those who are permanently disabled. Although they may experience the frustrations of the disability, they do not fully enter the world of people with disabilities because they know they will soon reenter society as able bodied. In contrast, those who develop a permanent disability (e.g., as in a CVA) must learn to incorporate the modifications required for living into their daily living and identity. DIF: Cognitive level: Analysis REF: Page 417

Jada understands that mental illness is one major risk factor for suicide. If she were to plan a presentation for high school teachers, she would discuss other risk factors for suicide, which include which of the following? Select all that apply. A. Substance abuse B. 15- to 24-year-old age group C. Individuals threatening to hurt themselves D. Previous history of a suicide attempt

A. Substance abuse B. 15- to 24-year-old age group C. Individuals threatening to hurt themselves Suicide is the third leading cause of death among those aged 15 to 24 years. If a person has a history of a previous suicide attempt, that does not mean that person is at risk of attempting suicide at any given moment. Warning signs that indicate acute risk for suicidality may be observed in individuals who are threatening to hurt themselves, alcohol abuse, and other substance use disorders.

Based on the social justice model, which intervention would the county health department be most likely to implement? A. Provide free sexually transmitted disease (STD) testing at the shelter B. Organize a job fair at the shelter C. Coordinate an Alcoholics Anonymous (AA) meeting at the shelter D. Encourage those living at the shelter to find permanent housing

A. Provide free sexually transmitted disease (STD) testing at the shelter Providing free STD testing at the shelter would be an intervention the county health department would be most likely to implement based on the social justice model. Under the social justice model, all people are equally entitled to key ends. Thus, all members of society must accept collective burdens to provide a fair distribution of these ends. The county health department would be incurring the costs of free STD testing, which would be paid for through tax dollars.

What is the most important adolescent developmental characteristic to take into account when the group is identifying interventions targeted to adolescent health concerns? A. Risk-taking behavior B. Growth in abstract thinking C. Increased interest in physical appearance D. Secondary sex characteristics appear

A. Risk-taking behavior Risk-taking behavior is responsible for many of the health risks seen in adolescents, including teen pregnancy, substance use, unintended injuries (including motor vehicle accidents), and violence.

Mr. K, age 35 years, is in the nursing clinic for a follow-up health visit. Additional gender-specific educational information should be offered about which of the following subjects? A. Seat belt usage B. Falls C. Digital rectal exams D. Immunizations

A. Seat belt usage Men remain three times less likely to use a seat belt as women of the same age despite aggressive legislative action for seatbelt usage. The other options are not gender specific.DIF: Cognitive level: KnowledgeREF: Page 343

What information should the nursing students share about advance directives? A. The Patient Self-Determination Act requires health care facilities to inform patients in writing about their rights to execute advance directives. B. A durable power of attorney is a legal document whose purpose is to allow individuals to specify what type of medical treatment they want to receive if they are incapacitated. C. A durable power of attorney for health care authorizes someone to act on an individual's behalf with regard to property and financial matters. D. A living will allows an individual to designate a health care proxy or surrogate to make decisions about medical care if one is unable to do so.

A. The Patient Self-Determination Act requires health care facilities to inform patients in writing about their rights to execute advance directives. The Patient Self-Determination Act requires health care facilities to inform patients in writing about their rights to execute advance directives. This is a federal law that applies to facilities that receive Medicare and Medicaid funds. A living will is a legal document whose purpose is to allow individuals to specify what type of medical treatment they want to receive if they are incapacitated. A durable power of attorney authorizes someone to act on an individual's behalf with regard to property and financial matters. A durable power of attorney for health care allows an individual to designate a health care proxy or surrogate to make decisions about medical care if one is unable to do so.

A nurse eagerly explained to a long-term disabled male client the most recent relevant research related to his care. The client, rather than express appreciation, said, "I prefer to have my care given this way." The nurse explained that research suggested an alternative approach was more clinically effective, but the client said, "I am more comfortable during the day if my care is given this way." Which of the following actions should be taken by the nurse? a. Accept that the client knows what works best b. Ask the nursing supervisor to review the research with the client c. Give evidence-based care according to current research findings d. Negotiate a compromise between patient preferences and research-based approach to care

ANS: A A person who lives with a disability commonly becomes an expert at knowing what works best for his or her body. The nurse must ask the client what works best for him or her and what goals the client is pursuing. Research is based on probabilities of effectiveness with large groups, but individuals may have a different unique reaction to any intervention.

Which of the following describes what has been identified by public health experts as the number one priority in rural areas? a. Access to health care b. Access to education c. Access to telecommunications d. Access to better-paying employment opportunities

ANS: A Bolin and Gamm (2003) reported that access to health care was the number one priority identified by the majority (73%) of rural health care leaders. Although access to education, telecommunications, and well-paying employment are important, they have not been identified as the number one priority in rural areas.

Which of the following best explains why companies do not build work places in rural areas where people are anxious to find full-time employment? a. Employers need highly educated and skilled employees. b. Managers do not want to be transferred into a rural area where they will not fit in and will not be comfortable. c. Rural persons do not work as hard as urban dwellers. d. Smells emanating from farms are unacceptable to persons working inside.

ANS: A Counties that have a low-wage economy have difficulty providing the infrastructure needed to provide education for low-wage families. They therefore have difficulty attracting new employers who might contribute to the economic development of a rural area but need a more highly educated workforce. Rural workers are known for their strong work ethic, but they may not have the educational skills that are necessary. Smells from farms is not a typical reason as to why companies do not build in rural areas.

A nurse was becoming very frustrated at the ongoing stream of farmworkers with respiratory problems each spring. Which of the following actions should be taken by the nurse? a. Provide community education on herbicides and other chemicals spread each spring with information on self-protective measures, such as respirators. b. Fight to get farmers to quit spreading chemicals on their fields each spring. c. Suggest that farmers spray water on the fields immediately after the chemicals are spread to reduce dust in the air. d. Tell farm workers to obtain different employment. e. Try to get farmers to reduce the amount of chemicals used on their fields.

ANS: A Environmental health issues in rural communities, such as pesticide exposure or health hazards from point-source factory emissions, are more effectively assessed and remedied on a community level than on a case-by-case basis. Nurses' involvement in helping people understand health problems in a larger context can be the genesis of change. It is not realistic to have farmers quit or reduce the spreading of chemicals, spray water on their fields after chemical application, or ask them to obtain different employment. Nurses must work with farmers so that they are able to use safe measures when applying the chemicals to their fields.

Which of the following is most likely to provide public health services to Americans residing in a rural community? a. Public health nurse b. Public health physician c. Epidemiologist d. Nurse practitioner

ANS: A In a study of 99 local health departments in three rural states, Rosenblatt and colleagues (2002) found that public health nurses were the core provider of public health services. Many smaller local health departments do not have the economic base to employ other professional public health providers, such as physicians, epidemiologists, sanitarians, and nutritionists at the local level.

As a concerned citizen and professional, a nurse encourages the local senators and representatives to support expansion of health clinics for the poverty stricken. Which of the following locations is in most desperate need? a. Rural areas where migrant and seasonal farmworkers (MSFW) labor b. Inner city poverty-stricken neighborhoods c. Near elderly low-income housing units d. Inside high schools in poverty-stricken neighborhoods

ANS: A In general, MSFW may have the poorest health of any aggregate in the United States and the least access to affordable health care. Thus, a location that serves this population would be most important.

Which of the following concerns can a nurse as a citizen help address? a. Advocating for removal of environmental and social barriers to needed services b. Continuing to engage in political action to increase funding for care for the handicapped c. Recognizing that it is better to offer to help than to wait for a person with handicaps to have to ask for assistance d. Needing to not stare or ask personal questions of a person with a disability

ANS: A In the National Agenda for the Prevention of Disabilities model, disability occurs when a person's physical or mental limitations, in interaction with physical and social barriers in the environment, prevent the person from taking equal part in the normal life of the community. Confronting environmental and social barriers to needed services can frustrate and exhaust many people with disabilities and their families. The nurse—as a citizen and a concerned professional—can advocate for removal of such barriers. This issue is the most important for the nurse to address, allowing the nurse to partner with clients and families affected by disabilities to remedy barriers that negatively affect quality of life for this population.

4. Which of the following insights is being stressed by advocacy groups regarding persons with disabilities today? a. Disability results when physical and social barriers in the environment prevent a person from taking equal part in community life. b. Handicapped persons have as much right to social support as nonhandicapped persons. c. It is recognized that an individual may be limited by an environmental barrier in society. d. Impairment is a result of society's reaction more than mental or physical condition.

ANS: A Nearly all definitions identify an individual as disabled on the basis of a physical or mental impairment that limits the person's ability to perform an important activity. The complementary possibility—that the individual is limited by a barrier in society or the environment—is never considered. Advocacy groups are stressing that environment and the reactions of others are as crucial as the actual physical or mental limitation.

A man came for follow-up care to the free clinic, explaining that he had applied for a job based on his education and years of previous work experience, but, being in a wheelchair, he was turned down. Which of the following responses would be most appropriate for the nurse? a. "If you know you can do the job, go to the free legal aid clinic for assistance. The employer may not know the law." b. "No one would want to hire a handicapped person, so you can continue to come here for free care." c. "Because it can be expensive to obtain the necessary equipment to help you do that job, the employer has every right to refuse to hire you." d. "The law says, if a handicapped person applies, he or she has to get hired even if an able-bodied person also applies."

ANS: A The Americans with Disabilities Act (ADA) became law in 1990. This landmark civil rights legislation prohibits discrimination toward people with disabilities in everyday activities. The ADA guarantees equal opportunities for people with disabilities related to employment, transportation, public accommodations, public services, and telecommunications. A qualified individual with a disability must meet legitimate skill, experience, education, or other requirements of an employment position. The person must be able to perform the essential functions of the job.

Which of the following was suggested by the National Healthcare Disparities Report of 2008 (NHDR) as one reason that there are such disparities in health care outcomes inside the health care system? a. Care provider bias and poor communication exist. b. Funding is inadequate to give appropriate care to persons with disabilities. c. Many people do not seek care until it is almost too late. d. There is a national shortage of care providers.

ANS: A The NHDR explains that, within the scope of health care delivery, disparities are due to differences in access to care, provider biases, poor provider-patient communication, poor health literacy, and other factors. This report did not address that there is inadequate funding to provide appropriate care, that people seek care too late, or that there is a national shortage of providers.

Which of the following best describes the problem with the viewing disability based on the Nagi model? a. Functional limitations are used to determine if an individual is disabled. b. Functional limitations are not easily objectified and measured. c. The model does not distinguish between genetic limitations and limitations imposed by injury or illness. d. Persons are or are not limited according to choices they make.

ANS: A The Nagi model uses functional limitations, which have been defined and are measureable, to determine whether an individual is disabled. However, the current paradigm considers the environmental barriers and the perspectives of the onlooker when considering if an individual is disabled. Thus, the other options do not address the limitation of using only functional limitations.

Which of the following persons would be called chronically homeless? a. A person has been homeless for more than a year b. A person who lives with a roommate when between jobs c. A person who has been staying with friends or family for an extended period of time d. A person has been residing each night in a homeless shelter for almost 3 months

ANS: A The chronically homeless individual is defined as an unaccompanied adult who has been homeless for an extended or numerous periods and has one or more disabling conditions.

Which of the following best describes the practical importance of the classifications of metropolitan, micropolitan, and noncare areas? a. Classifications are used for congressional policy-making and funding decisions. b. Metropolitan areas gained importance because of their large population. c. Micropolitan areas are now recognized for their crucial role in society. d. The new system emphasizes the extent of rural (noncare) areas.

ANS: A The new core-based statistical areas (CBSA) system includes two categories of counties: (1) metropolitan areas and (2) micropolitan areas. Counties that are neither metropolitan nor micropolitan are called "outside CBSAs," also known as noncare areas. Metropolitan areas contain at least one urbanized area of 50,000 or more people. A micropolitan area contains a cluster of 10,000 to 50,000 persons. Because the metropolitan statistical area classification is used extensively for congressional policy-making and funding decisions, there could be serious ramifications for health care financing within rural market areas.

Which of the following underlying factors most likely contributes to homelessness? a. Substance abuse b. Unemployment c. Posttraumatic stress syndrome (PTSD) d. Severe mental illness

ANS: A The text states that persons as active agents do make decisions that result in homelessness. These choices result in a shortage of affordable housing, insufficient income, and scarcity of supportive services as societal conditions that contribute to homelessness rather than cause homelessness. However, these decisions are made in highly contextualized conditions, frequently addiction to alcohol and/or other substances. Substance abuse may in turn lead to unemployment which causes insufficient income. Additionally, PTSD and severe mental illness can cause substance abuse to occur.

Which of the following compositional factors influences rural health needs? a. Demographics of the people living in an area b. Local environment c. The geography of the area d. The political and social institutions in the community

ANS: A There is a growing emphasis on the distinction between context, which is defined by the characteristics of places of residence, and composition, which is the collective health effects that result from a concentration of persons with certain characteristics. Problems in rural areas are compositional when they derive from individual characteristics of groups of people residing in rural settings. Examples of compositional sources of health disparities include such characteristics as age, education, income, ethnicity, and health behaviors. The environment, geography, and political and social institutions are considered to be characteristics of place.

A nurse was trying to decrease the amount of respiratory illness in the community. Which of the following approaches should be used by the nurse? a. Attend each farmworkers' meeting and listen to how they perceive and discuss their problems b. Offer to teach the farmers' spouses how to protect their families, such as having the farmer shower and change clothes before coming into the house c. Suggest the community work together to obtain a community college so farmworkers could be educated for other employment opportunities d. Teach children in the school system the importance of seeking employment in cities rather than staying on the farm

ANS: A Using informal networks in the community is essential. The involvement of informal networks and local leaders in planning health interventions is crucial. To be able to do that, the nurse must first listen to how the community perceives its problems.

Which of the following factors results in rural residents having less access to health care than urban dwellers? (Select all that apply.) a. Employment opportunities may be low paying and seasonal. b. There are greater distances to travel and limited choice of health care providers. c. Health care providers prefer to work in higher-paying urban settings. d. Rural residents are less likely to have health insurance. e. Rural persons' economic resources are tied up in land, not easily accessible cash. f. Rural persons believe in self-sufficiency and self-care, not asking others for help.

ANS: A, B, C, D A recent Institute of Medicine study reported rural medical access problems in these areas, with some hospital and pharmacy closures; greater distances to travel for physician services; and limited, if any, choice of providers. Rural residents were more likely than urban residents to lack insurance. Two thirds of the persons living in the most rural counties are low-income families. Having economic resources invested in land and believing in self-sufficiency and self-care are not reasons why rural residents have less access to health care than urban dwellers.

Which of the following do expert rural nurses report as benefits of their work? (Select all that apply.) a. Ability to develop close relationships and give holistic care b. Being able to practice in the community where one lives c. Constant challenges d. Continual growth as an expert generalist e. Higher salary than in urban areas f. Intensity of purpose

ANS: A, B, C, D, F Positive aspects of rural nursing include the ability to give holistic care, to know everyone well, and to develop close relationships with the community and with coworkers. Autonomy, professional status, and being valued by the agency and community have been reported components of positive job satisfaction. The rural nurse is a generalist, and generalist is not synonymous with boring. Interviews with rural nurses show that they feel an "intensity of purpose" that makes rural nursing distinctive. Nurses living and practicing in the same place have a strong sense of integration and continuity between practice and community.

Which of the following best describes why for-profit managed care organizations (MCOs) have not begun to move into rural areas to increase the number of persons served? (Select all that apply.) a. High startup and administrative costs are needed. b. A large number of elderly are on Medicare or Medicaid, which results in low reimbursement. c. Managed care has not yet had time to go beyond major population centers. d. MCOs are waiting until federal reimbursements for underserved rural health areas become higher. e. The population is too small and too spread out. f. Rural health areas are too geographically unattractive and lack any recreational possibilities for managers to want to work there.

ANS: A, B, C, E MCOs recognize the risks to expanding into rural areas. Risks are probable high startup and administrative costs. Managed care has yet to move into much of rural America because of small dispersed populations, few visits per individual, and large numbers of elderly on Medicare with low-level reimbursements that do not make the aggregate financially attractive to MCOs. It is not likely that federal reimbursements for underserved rural health areas will become higher. The lack of attractive living for managers has not been a reason why MCOs have not moved into rural areas.

Which of the following best describes the term "literally homeless"? (Select all that apply.) a. Lacking a fixed, regular, and adequate nighttime residence b. Lacking a family residence c. Living in a place not typically used for human sleeping d. Living in a temporary living accommodation such as a hotel e. Living full-time in a recreational vehicle f. Sleeping in an extended-care facility or hospital

ANS: A, C Literally homeless is described as individuals and families who lack a fixed, regular, and adequate nighttime residence and includes a subset for an individual who resided in an emergency shelter or a place not meant for human habitation and who is exiting an institution where he or she temporarily resided. The term "literally homeless" is not used to describe those lacking a family residence, living in a temporary living accommodation such as a hotel or recreational vehicle, or sleeping in an extended care facility or hospital.

Which of the following best describes why there are more long-term consequences (such as death) to a driver injured in a truck accident in a rural area than a driver injured in a truck accident in an urban area? (Select all that apply.) a. Rural drivers are more likely to speed over long distances than urban drivers. b. Rural drivers are not as skilled with vehicles as urban drivers, so they are more severely injured in a crash. c. The urban driver is taken immediately for care, whereas the rural driver might be injured on an isolated field and not found by anyone for hours. d. Urban drivers are more likely to use their seat belts than rural drivers. e. Urban drivers are probably rushed to an emergency department, whereas the rural driver lives far away from emergency services, often over bad roads and possibly through severe weather. f. Urban drivers trade their vehicles in rather frequently, whereas rural drivers keep their vehicles for as long as they can make them run, so the vehicles are much less roadworthy.

ANS: A, C, D, E Getting patients from the place of injury to the trauma center within the "golden hour" is frequently not possible in rural areas because distance, terrain, climatic conditions, and communication methods produce barriers. Some rural facilities are more than 1 hour away by air from the nearest trauma center or tertiary care hospital. Driving at high speeds, driving long distances, driving in winter conditions, not using seat belts, and consuming alcohol have been cited as contributing to greater levels of injury deaths and disability by rural residents in the West. There is no evidence to suggest that rural drivers are not as skilled as urban drivers or that their vehicles are less roadworthy.

In comparison with mental health care in urban areas, which of the following best describes how mental health care in rural areas differs? (Select all that apply.) a. Access to care is more limited or not available at all locally. b. Cost for treatment is typically reimbursed at a lower rate. c. Depression is frequently not recognized or is not treated. d. Ethnic and racial disparities exist. e. Primary health care professionals accept responsibility for mental health treatment. f. The suicide rate is notably higher.

ANS: A, C, E, F In rural areas adjacent to a small city, suicide rates were 31% higher than suburban rates and 43% higher in rural areas that were not adjacent to small cities. Decreased access to mental health services for treatment of depression may contribute to these higher rates. Because of the lack of qualified mental health care providers, rural residents often receive services from primary care providers. Ethnic and racial disparities exist whether in urban or rural health care.

Which of the following guiding rules or principles would most likely be followed by a nurse caring for persons with disabilities? (Select all that apply.) a. Adopt the client's priorities b. Change care according to the most recent research findings c. Care for the disability and then for the whole person d. Know community resources thoroughly e. Support the client's decisions f. When in doubt, listen

ANS: A, D, E, F Data from interviews conducted with nurses who provide care to persons with disabilities revealed principles such as do not assume anything; adopt the client's perspective; listen and learn from the client; gather data from the perspective of the client and family; care for the client and the family, not the disability; be well informed about community resources; and become an advocate by letting clients choose.

A nurse was excited about being the only staff member in the public health department in a small rural area. Which of the following problems would the nurse most likely encounter? (Select all that apply.) a. There is no one with whom to collaborate or exchange ideas. b. There will be no physician available to tell the nurse what needs to be done. c. No feedback can be obtained from others. d. The nurse may only be able to offer services supported by federal funds. e. The nurse will have to raise local funds to support the department. f. There will not be any epidemiologists or sanitarians to help.

ANS: A, D, F Many smaller local health departments do not have the economic base to employ other professional public health providers, such as physicians, epidemiologists, sanitarians, and nutritionists at the local level. The lack of additional onsite public health providers poses two problems. First, the ability to collaborate with others about potential or actual public health problems is decreased or nonexistent. Second, the ranges of services provided are less comprehensive. Often, these small facilities can only offer federally funded programs with few locally funded services. The nurse should be able to think independently to determine what needs to be done and should be able to work without physician direction. Feedback can be obtained from the population that the nurse is serving. Local, state and federal funds may all support the services that the nurse is providing. It is unlikely that the nurse will need to raise funds to support the work of the department; there should be local funding available for this nurse's position.

Which of the following best describes the current perspective on how communities should treat persons with disabilities (PWDs)? a. Be supportive of PWDs and try to reduce environmental barriers b. Maximize opportunities for PWDs to work and otherwise contribute to community life c. PWDs have civil rights just as other citizens do. d. Recognize that PWDs deserve our support both financially and otherwise

ANS: B Contemporary disability policy minimizes disadvantages and maximizes opportunities for PWDs to live productively in their communities. Early American public policy viewed PWDs as "deserving poor" who required governmental protection and provision, with little capacity for self-support or independence.

Which of the following describes a dimension of the social determinants of health (SDH) defined within Healthy People 2020? a. Employment b. Education c. Gender d. Income

ANS: B Five broad dimensions of SDH are defined within Healthy People 2020 as: (a) Economic Stability, (b) Education, (c) Social and Community Context, (d) Health and Health Care, and (e) Neighborhood and Built Environment. Each of these dimensions includes key issues that identify more specific factors (U.S. Department Health and Human Services Healthy People 2020, 2013). Employment, gender, and income are not addressed as dimensions of the social determinants of health within Healthy People 2020.

Which of the following statements best describes how firearm injuries are different in rural areas as compared with urban areas? a. In rural areas, firearm deaths occur more frequently because of purposeful, rather than accidental, shooting. b. In rural areas, firearm injuries occur more frequently in the home. c. In rural areas, firearm injuries occur more frequently during official hunting season, whereas the frequency does not vary in urban areas. d. In rural areas, firearm injuries are more frequently fatal than in urban areas.

ANS: B In rural counties, nonfatal firearm injuries occur most often at home compared with urban counties, where injuries occur most often in the streets.

A professional strongly encouraged institutionalization so the parents could focus on care of their other family members. Which of the following suggestions would be appropriate for the nurse to make to the parents? a. As the expert pointed out, institutionalization will reduce family stress. b. The child has a right to an education in the least-restrictive setting. c. The child will eventually adapt to whatever choice the parents make. d. The choice of what to do is entirely the parents'.

ANS: B It is a given that it is the parents' choice. As an advocate for the child, you can explain that children have a legal right to an appropriate public education based on the child's needs in the least-restrictive setting.

Which of the following facts is noteworthy about adolescent males who are driving without a license in the countryside? a. Most had driver's education classes but have not yet earned the money to pay for their driver's license test. b. Most were Hispanic or black from a rural area. c. Most were practiced in driving large farm machinery and were just going from one section of the farm to another section. d. Most were practicing their driving skills with their parent in the car.

ANS: B Researchers studied the prevalence of unlicensed teenaged drivers compared with licensed drivers and found that they were more likely black or Hispanic and live in rural areas. There is not any research in the text that supports that the other responses are accurate.

A nurse came limping into work with a huge cast on her right leg and asked to have an assignment for a week or two that did not require driving. The nurse said, "I never realized how difficult it is to get some places without two good legs. Now I know how persons with disabilities must feel!" Which of the following would be the most appropriate response from the nursing supervisor? a. "I'm sure you'll be even more understanding and supportive now." b. "Not really; you'll be out of that cast in a few weeks." c. "What a great opportunity to attend some support groups for people with disabilities and share with us which ones are most helpful." d. "Why don't you just take some sick leave rather than try to work with that leg cast?"

ANS: B Those who have a temporary disability have a very different experience than those who are permanently disabled. Although they may experience frustrations, they view it as a temporary problem and a temporary setback. The other responses address this temporary disability as if it is similar to a permanent disability, which it clearly is not.

Which of the following best describes how the Continuum of Care (CoC) concept determines the number of homeless? (Select all that apply.) a. By asking local police departments to visit areas where the homeless hide and report the number of people found there each March b. By conducting a point-in-time (PIT) homeless persons count every other January c. By making an educated guess based on anecdotal data d. By completing a statistical analysis of the Census Bureau data e. By reporting the number of homeless persons who use emergency shelters or transitional housing during the year f. By sending CoC personnel into the community to try to find hidden homeless

ANS: B, E CoCs are local systems responsible for providing a range of housing and related services that meet the U.S. Department of Housing and Urban Development guidelines for persons experiencing homelessness. CoCs conduct PIT counts of homeless persons on one night in January of every other year. In addition, CoCs are required to report the number of homeless persons who use emergency shelters or transitional housing throughout each federal fiscal year.

Which of the following best explains why there are so few hospitals in rural areas? a. Few nurses and physicians enjoy the rural lifestyle. b. Historically, hospitals began in large urban areas and have slowly expanded outward but have not yet reached all rural areas. c. Rural areas have very low population density. d. Rural residents do not trust hospitals, preferring to receive care from neighbors and friends.

ANS: C Although 75% of U.S. counties are classified as rural, they contain only 20% of the U.S. population. The population density is too low to support a large hospital. There are providers available to serve the population and residents are accepting of health care services, but not as many services are needed because of the lower population density.

Which of the following best describes why some advocacy groups have taken a strong stand against physician-assisted suicide? a. Ethical belief that life is always preferable to death b. Emphasis that there is always hope for a research breakthrough or discovery of a new treatment intervention c. Fear that some persons may be "encouraged" to accept an early death merely because they are not valued by society for their contributions d. Helping someone die is murder, plain and simple

ANS: C Because people have previously been killed because they had a disability, some advocacy groups, such as Not Dead Yet, have taken a strong stance against physician-assisted suicide, fearing it will lead to the early or forced death of people with disability. Ethical and personal beliefs and emphasizing the hope for a new treatment have not been stands taken by advocacy groups against physician-assisted suicide.

Which of the following statements best describes how working on a farm is different from working in a factory? a. Factories have more dangerous dust than farming dust. b. Farm chemicals are more immediately fatal than most industrial chemicals. c. The worksite and the home are the same. d. Workers on farms, unlike those in factories, can bring dangerous substances home on their clothing, which expose their families.

ANS: C Farming is unusual because the home and the worksite are the same. Several types of farming activities are associated with higher-than-expected occurrences of acute and chronic respiratory conditions. Individuals with long-term exposure to grain dusts, such as grain elevator workers and dairy workers, have diminished respiratory function and increased frequency of respiratory symptoms. Occupational asthma and more exotic fungal or toxic gas-related conditions occur in higher frequency in agricultural than nonagricultural populations. Exposure risks to children and spouses may be heightened when farmers wear contaminated clothing and boots into the home, but the same can happen in several industries.

Which of the following persons will have the most difficulty in obtaining adequate support services to find a long-term home? a. An alcoholic homeless person b. A homeless person who has a limited support system and is unemployed c. A homeless person who has a mental illness and is an alcoholic d. A mentally ill homeless person

ANS: C For a sizable proportion of the homeless, severe mental illness exists along with the problems of alcohol or other types of substance use. Like physical health problems, serious mental illnesses and minor emotional problems occur more frequently among the homeless population than in the general population. High rates of alcohol and drug use exacerbate the existing acute and chronic physical and mental health problems. These coexisting problems can make it harder to assist the homeless person

A school nurse was asked to see Lisa, a student in the fourth grade who was rumored to be living in a car with her mother. Which of the following would most likely have caused the teacher to become concerned? a. Lisa is not always respectful of the teacher. b. Lisa has not been doing her homework. c. Lisa may be developmentally delayed. d. Lisa may have behaved in a sexually inappropriate manner.

ANS: C Homeless children have higher rates of physical, mental health and behavior, and educational problems than children in the general population. All children are not always respectful and do not always have their homework done and may act out behaviors seen on television.

Which of the following best describes the purpose of the Vulnerability Index? a. To identify those most at risk of becoming homeless b. To determine the underlying cause of homelessness among vulnerable populations c. To identify and prioritize the need for housing among the homeless d. To determine what resources should be provided for vulnerable populations

ANS: C Homeless service providers, concerned about the high risk for mortality among "street" homeless population constructed the Vulnerability Index, a screening tool for identifying and prioritizing the need for housing.

The male home health nurse parked his marked car in preparation for visiting the next client when a young female walked over to the car and asked him if he would like to have some fun. Which of the following provides the best explanation for this behavior? a. The girl was skipping school and wanted money for entertainment. b. The male nurse was extremely young and attractive and looked like he had money. c. The girl may have hoped a nurse would give her enough money to eat. d. Young girls often engage in thrilling, if high-risk, behaviors.

ANS: C Homeless youth experience sexually-transmitted diseases, physical and sexual abuse, skin disorders, anemia, drug and alcohol abuse, and unintentional injuries at higher rates than in the general population. Depression, suicidal ideation, and disorders of behavior, personality, or thought also occur at higher rates among homeless. Family disruption, school failures, prostitution or "survival sex," and involvement with the legal system indicate that homeless social health is severely compromised. Thus, it is most likely that the girl is hoping that the nurse would provide her enough money to eat.

Which of the following best describes what happens when persons who work full-time and receive the typical minimum wage seek an apartment? a. Few clean and decent low-rent apartments exist. b. People do not like to rent to low-income persons. c. To afford housing, they need a job that pays at least three times minimum wage. d. There are not many trailer parks near their employment.

ANS: C In many areas of the country, wages needed to afford housing are three to five times higher than the federal minimum wage of $7.25 per hour.

Which of the following best describes a major difference between women who have always had shelter and women who have been homeless? a. Educational opportunities and attainment b. History of gender relationships (marriages, divorces) c. Level of involvement with physical and/or sexual abuse d. Skills necessary for paid employment

ANS: C In the 1990s, research clearly documented the extraordinary histories of violence, from childhood through adulthood, among women experiencing homelessness. High numbers of homeless women report histories of foster placement as children and exposure to violence as both children and adults. This research has not documented the differences in educational opportunities, gender relationships, and skills necessary for paid employment.

Which of the following statements best reflects the model of justice U.S. health care is based on? a. All people are entitled to minimum standards of care. b. All people are responsible to help their neighbors. c. All people have a right to whatever their own efforts allow them to purchase. d. Like firefighting, police protection, libraries, and roads and highways, health care must be supported by tax money for the good of all who need such services.

ANS: C Market justice has been the dominant model and purports that people are entitled to valued ends (i.e., status, income, and happiness) according to their own individual efforts. Moreover, this model stresses individual responsibility, minimal collective action, and freedom from collective obligations other than respect for another person's fundamental rights. In contrast, under a social justice model, all people are equally entitled to key ends (i.e., access to health care and minimum standards of income). Consequently, all members of society must accept collective burdens to provide a fair distribution of these ends.

Which of the following are particularly prevalent in rural areas (more so than in urban areas) of the South? a. Falls and motor vehicle accidents b. Less use of seatbelts and more drinking of alcohol c. Obesity and sedentary lifestyle d. Suicide and homicide

ANS: C Rural residents in the southern states are more likely to be obese, smoke more heavily if they do smoke, use smokeless tobacco, and engage in sedentary lifestyles. Rates of motor vehicle accidents and suicide are higher in rural areas, although not necessarily the South. Additionally, there is a decreased use of seatbelts and increased use of alcohol in rural areas; however, the textbook mentions that these rates are higher in the rural West than in the rural South. The rates of falls and homicide are not addressed in the textbook.

Which of the following groups of people is most likely to experience homelessness? a. Those who have a supportive family b. Those who have recently become unemployed c. Those who have substance abuse problems d. Those who frequently change jobs

ANS: C Some people experiencing homelessness have individual characteristics that, in interaction with the structural conditions of a shortage of affordable housing and insufficient income, perpetuate their homeless conditions. Supportive services for these people are deficient in quality and quantity. Some people need services to work and earn money. They are able to function in the workforce, whereas others need services to maintain their housing status. Included in this latter group are people whose serious chronic mental health and/or substance abuse problems preclude their functioning in the workforce and whose behaviors interfere frequently with their ability to obtain housing stability. People in this group need income assistance and comprehensive and accessible behavioral and physical health care.

Which of the following best describes how the federal government tries to both encourage and support rural health clinics under the Community Health Centers (CHC) program? a. Allowing tax deductions for charitable contributions to rural health clinics (RHCs) b. Donating equipment and supplies to establish such clinics c. Reimbursing both Medicare and Medicaid at a higher rate than otherwise d. Requiring recipients to serve in a rural health clinic for 3 years in exchange for financial assistance while in school

ANS: C The CHC program, administered by the U.S. Public Health Service, benefits underserved areas and populations. RHCs are designed to improve access to primary care. As an incentive to rural communities to apply for RHCs, Medicare and Medicaid are reimbursed at a higher rate than usual.

Which of the following best describes how Migrant Health Centers (MHCs) under the Migrant Health Program are often different from other clinics? a. Care is restricted to those who can document that they are employed in agriculture. b. They are much better funded than typical rural clinics. c. Staff are typically bilingual and bicultural. d. They welcome illegal immigrants to receive care.

ANS: C The MHCs and the Migrant Health Program provide comprehensive nursing and medical care and support services to migrant and seasonal farm workers and their families from many countries in Latin America and the Caribbean. Bilingual, bicultural health personnel, including lay outreach workers, use culturally appropriate protocols for providing primary care, preventive health care, transportation, dental care, pharmaceuticals, and environmental health. The funding at the MHC may not be any different than other rural clinics. Their primary purpose is not to provide care to illegal immigrants, rather to migrant and seasonal workers.

A family will most likely lose their primary nighttime residence within the next week. Which of the following terms best describes their situation in relation to homelessness? a. Homeless under other federal statutes b. Literally homeless c. Imminent risk of homelessness d. Fleeing/attempting to flee domestic violence (DV)

ANS: C The U.S. Department of Housing and Urban Development has issued regulations that summarized the statutory definitions in four descriptive categories. These four categories with their respective definitions include: Literally homeless: Individuals and families who lack a fixed, regular, and adequate nighttime residence, including a subset for individuals who resided in an emergency shelter or a place not meant for human habitation and who are exiting an institution where he or she temporarily resided; Imminent risk of homeless: Individuals and families who will imminently lose their primary nighttime residence; Homeless under other federal statutes: Unaccompanied youth and families with children and youth who are defined as homeless under other federal statutes who do not otherwise qualify as homeless under this definition; and Fleeing/attempting to flee DV: Individuals and families who are fleeing, or are attempting to flee, DV, dating violence, sexual assault, stalking, or other dangerous or life-threatening conditions that relate to violence against the individual or a family member.

Which of the following best describes the recent trends in homelessness? a. The number of homeless individuals in school is decreasing. b. The number of homeless individuals is increasing. c. The number of people in homeless families is increasing. d. The number of people in homeless families is decreasing.

ANS: C The U.S. Department of Housing and Urban Development reported the following numbers of people in the total population and subpopulations who used shelters or transitional housing between October 1, 2010, and September 30, 2011: 4,469 individuals, a decrease of 11% since 2007; 537,414 people in families, an increase of 13.5% since 2007. According to the U.S. Department of Education: 1,065,794 homeless students were enrolled during the 2010-2011 school year an increase of 11% over the 3-year period 2008-2009 to 2010-2011.

Which of the following factors are most likely to contribute to homelessness? a. Domestic violence and substance abuse b. Scarcity of support systems and recent job loss c. Shortage of affordable housing and insufficient income d. Substance abuse and serious mental illness

ANS: C The text addresses three broad factors: (1) shortage of affordable housing, (2) insufficient income, and (3) scarcity of supportive services as societal conditions that contribute to homelessness rather than cause homelessness. Domestic abuse, substance abuse, and serious mental illness are not identified as the broad factor contributing to homelessness.

Which of the following factors would prevent a homeless person from accepting employment adequate to pay for housing? a. Could not complete an employment application because of lack of mailing address b. Hope for a better position than the service positions available c. Lack of adequate resources to pay for other necessities d. Prefer to receive a handout than to have to work

ANS: C There are homeless people who are able to function in the workforce. The lack of affordable housing in combination with insufficient income results in people having to spend much of their income on rent and leaves them without adequate resources for other necessities, such as food, clothing, and health care. This situation substantially increases their risk for homelessness.

A family has recently learned that their child will be permanently disabled. The parents do not deny the disability, but do not fully realize its impact. Which of the following levels of adjustment are the parents experiencing? a. Self-actualization b. Normalization c. Ostrich phase d. Special designation

ANS: C Ulrich and Bauer (2003) propose that the adjustment to disability experience occurs in four levels as parents gradually become aware of the impact of their child's disability. These levels include: (1) the ostrich phase, where parents do not deny a disability, but do not fully realize its impact; (2) special designation, when parents begin to realize their child has a special need and seek help; (3) normalization, when parents try to make the differences between their child and children without disabilities less apparent, and may actually request a decrease in services; and (4) self-actualization, when parents do not view being different as better or worse, just different, and support their child in learning about his or her disability, along with how to be a self-advocate.

Which of the following strategies have been used to improve the collection of prevalence data for the homeless population? (Select all that apply.) a. Urban Institute study b. Homeless Assistance Report c. Continuum of Care (CoC) concept d. Homeless Information Management System (HMIS) e. Census Bureau f. Education for Homeless Children and Youth program

ANS: C, D Two strategies used by the U.S. Department of Housing and Urban Development in the efforts to strengthen the nation's efforts to reduce homelessness, including improvement in collecting prevalence data, are the CoC concept and the HMIS. The Census Bureau and Urban Institute study were historically used to collect data on the homeless population. Data collection improvements have been made since these studies were completed. The Department of Education uses the Education for Homeless Children and Youth program to collect data on homeless youth.

Which of the following disorders is experienced at a higher rate by homeless adults when compared with the general population? (Select all that apply.) a. Hepatitis b. Diabetes c. Asthma d. HIV/AIDS e. Hypertension f. Alcoholism

ANS: C, E, F Chronic disorders experienced at higher rates than in the general population include hypertension; musculoskeletal disorder; gastrointestinal problems; respiratory problems (asthma, chronic bronchitis, emphysema); neurological disorders, including seizures; and poor dentition. Like physical health problems, serious mental illnesses and minor emotional problems occur more frequently among the homeless population than in the general population. High rates of alcohol and drug use exacerbate the existing acute and chronic physical and mental health problems. The rates of hepatitis, diabetes, and HIV/AIDS are not seen at a higher rate among the homeless adult population.

Which of the following factors would cause a geographic area to be called a frontier area? a. Arbitrary distinction recognizing the Great Plains, Intermountain areas, and Alaska b. Geographically located in the West, specifically the Great Plains states c. High death rate from use of guns (homicide or suicide) d. Population of fewer than six persons per square mile

ANS: D Areas are classified as rural for populations with fewer than 45 persons per square mile and as frontier for geographic areas with fewer than six people per square mile. Many counties of the Great Plains, Intermountain West, and Alaska are designated frontier. Not all mountain areas are rural (the Appalachian mountain area is not), and geographic location alone does not make an area rural.

Which of the following would be good rules to use when interacting with a person with disabilities? (Select all that apply.) a. Apologize for slips of the tongue such as saying "Do you see?" to a blind person b. Be friendly to guide dogs, including petting them and offering to get them fresh water c. Notice similarities between yourself and the person with a disability d. Take hold of a blind person's arm to assist them in dangerous situations, such as crossing a busy street e. Volunteer the most recent research findings related to the person's disability f. With someone who is blind, announce who you are and why you are there when you enter the room

ANS: C, F Rules include do not offer expert advice or assistance based on what you think the person needs or can do; do not grab the arm of a person who is blind but let the person take your arm so the person does not lose his or her balance; do not pet a working dog; do seek out similarities and shared interests; do inform the person who is present; say goodbye when leaving; and treat the person very much like an able-bodied person.

Which of the following describes the most important need of every school-aged child with a disability? a. A teacher who understands the child's needs b. An advocate within the school system c. An assessment with comparison with developmental milestones d. An individualized education plan (IEP)

ANS: D A child should have all of the above items, an advocate in the school nurse, a teacher who understands, an assessment including comparison with developmental milestones, as all of these are components of an IEP with goals, cooperatively developed with the parents, to help the child succeed in school. The IEP describes the goals, as well as any special support needed to help achieve them. The IEP is the most comprehensive answer.

Which of the following best describes a proposed strategy to address the lack of health care professionals in rural communities? a. Advertise that there are more men than women in many rural areas to attract female nurses who want to be wives and mothers b. Offer free housing and office space to a professional who will come to live in the community c. Pay extremely high salaries to professionals willing to relocate d. Support local students obtaining an education while continuing to live at home

ANS: D A solution proposed for the shortage of health care providers is for rural communities to "grow their own." A rural community, a group of small communities, or a county could support local students attending college and recruit students currently attending professional schools. The students make a commitment to work in the community in return for monetary support for their educations.

Which of the following factors is strongly predictive for homelessness among women veterans? a. Having a previous criminal conviction b. Being a mother c. Having a high school education d. Being unmarried

ANS: D Additional research has demonstrated that being unemployed, disabled, or unmarried strongly predicts homelessness among women veterans. Having a high school education, being a mother, and having a previous criminal conviction are less predictive of homelessness than marital status.

Which of the following is the most frequent cause of farm fatalities and nonfatal injuries? a. Manure pits (methane poison) b. Power take-off augers c. Storage silos d. Tractors e. Very old and barely functioning trucks

ANS: D Agricultural machinery is the most common cause of fatalities and nonfatal injuries of U.S. agricultural workers, including on-farm fatalities among youth under 20 years of age (CDC, NIOSH Program, 2012). Tractor-related accidents, especially rollovers, are the most frequent causes of farm accidents and account for more than one fourth of farm fatalities.

A nurse is using an upstream intervention when working with the homeless population. Which of the following public health interventions will the nurse most likely use? a. Case management b. Outreach c. Surveillance d. Community organizing

ANS: D Although nurses may use all interventions at all three levels, community/public health nurses working more upstream, at the system level, employ collaboration, coalition building, community organizing, advocacy, social marketing, and policy development and enforcement. In contrast, community/public health nurses—working downstream with individuals, families, or groups—use surveillance, disease and other health event investigation, outreach, screening, case finding referral and follow-up, case management, delegated functions, health teaching, counseling, and consultation.

Which of the following best describes the problem with the Section 8 housing program? a. Direct access to an apartment in a public housing facility is faster and easier. b. Most renters prefer the program to make direct payment to apartment owners willing to rent to them. c. Section 8 housing is essentially slum housing, and no one wants to live there. d. Supply is vastly less than the demand.

ANS: D Although these programs are intended to alleviate housing problems for low-income renters, the demand for these assisted housing programs has far exceeded the supply. Section 8 housing does not provide direct access to an apartment in a public housing facility or provide payment directly from the program to apartment owners. Section 8 housing provides affordable housing through the U.S. Department of Housing and Urban Development, and is not slum housing.

Which of the following represents the largest group of homeless adults? a. Minorities b. Disabled c. Females d. Males

ANS: D Among all sheltered homeless adults, approximately 63% were men; 37% were women. Men were overrepresented compared with the general population. The sheltered homeless population is younger than the general population. Most homeless adults were between 31 and 61years of age; 22.1% were less than 18 years of age; and 3% were older than 62. In the general population, 16% were over age of 62. Minorities comprised 60% of the total sheltered population. More sheltered homeless people had disabilities than did members of the general population: 38.1% versus 15%.

Which of the following individuals would be described as being "doubled up"? a. An individual who has two chronic conditions b. An individual who has recently been released from prison c. An individual who maintains two jobs to avoid being homeless d. An individual who is forced to live with friends

ANS: D An individual may be considered to be homeless if that person is "doubled up," a term that refers to a situation where individuals are unable to maintain their housing situation and are forced to stay with a series of friends and/or extended family members. The other responses do not accurately describe being "doubled up".

Which of the following best describes a downstream intervention to solve the problem of homelessness? a. Improved use of community resources b. Increased employment opportunities c. Increased affordable housing d. Improved treatment of mental illness

ANS: D Building on McKinlay's "river" metaphor, McKinlay and Marceau (2000) purport that government and private efforts to address homeless health care problems largely focus on "pulling the bodies out of the river of homelessness." Such downstream interventions aimed at treating or alleviating health care problems, such as physical disease and mental illnesses, are worthy and needed. However, these interventions when used alone are far less adequate in alleviating homeless people's social health problems. To improve the social health of the homeless, it is necessary to go upstream and focus on the primary contributors to homelessness itself (i.e., lack of affordable housing, inadequate income, and insufficient services).

Which of the following are the most frequent problems suffered by the chronically homeless? a. Chronic conditions such as hypertension, bronchitis, and emphysema b. HIV/AIDS and other sexually transmitted diseases, tuberculosis, and hepatitis C c. Respiratory infections, trauma, and skin disorders d. Severe mental illness and substance abuse disorders

ANS: D Chronically homeless people may well have many physical conditions, but the primary problems that lead to their homelessness are severe mental illness and substance use disorders.

A nurse is explaining why migrant farmworkers experience cultural barriers when trying to receive care. Which of the following would the nurse include in this explanation? a. "As illegal immigrants, they are not eligible for tax-paid health care." b. "Families who travel in isolation seeking agricultural work do not mix with their employers or other members of the community." c. "Mexicans who speak primarily Spanish and cannot understand English." d. "Usually perceived as outsiders, they are from many countries and cultures."

ANS: D Cultural, linguistic, economic, and mobility barriers all contribute to the nature and magnitude of health problems observed in farmworkers. Cultural and linguistic barriers are the most overt because many of the communities where farmworkers work consider them outsiders. A common misconception among U.S. health care providers is that these farmworkers are from Mexico, with Spanish as their primary language. Farmworkers originate from many communities in Mexico, the Caribbean, and Central and South America, and they may speak English, the language of their home country, or several languages.

Which of the following was the result of the passage of the American Recovery and Reinvestment Act of 2009? a. A few employers accepted the extra funds and employed more people with disabilities (PWDs). b. Large numbers of employers accepted the extra funds and employed more PWDs. c. Employers still refused to employ PWDs. d. Little change has occurred.

ANS: D Employers willing to hire PWDs may receive monies provided through the American Recovery and Reinvestment Act (2009). However, little improvement in employment levels has been seen.

Which of the following best describes the proportion of the U.S. population that is made up of persons with a long-lasting condition or disability? a. The number is about 5% and increasing. b. The number is about 10% and stable. c. The proportion is about 15% and stable. d. The proportion is almost 20% and increasing.

ANS: D In 2010, approximately 56.7 million (18.7%) of the 303.9 million civilian noninstitutionalized population aged 5 years and older had a long-lasting condition or disability. Further, it is important for health care policy makers and health care providers to recognize that the prevalence of disability is increasing.

Which of the following best describes how health care professionals are involved in the problem of homelessness? a. Homeless persons make everyone uncomfortable when they beg for handouts. b. Homeless persons often live in emergency departments and clinics. c. Poverty leads to crimes, and health care professionals can be attacked and robbed outside health care facilities. d. Serious illnesses or disability creates high medical bills, which may lead to homelessness.

ANS: D In 2011, of households with an annual income less than $25,000, 25.4% had no health insurance. Lack of health insurance is a significant factor in creating homelessness. A serious illness or disability can lead to a downward spiral as a result of job loss, use of savings to pay for care, and inability to pay rent.

Which of the following differences is found in parish nursing in a rural area as opposed to parish nursing in an urban area? a. Rural parish nurses are less likely to be involved in case management or care coordination than their urban counterparts. b. Rural residents are more grateful for the nurse's assistance because there are so few health resources in rural areas. c. Rural residents are typically less committed to traditional values or a strong religious faith. d. Rural residents may discuss health concerns with the nurse wherever they meet, whether in church or the local market.

ANS: D In urban settings, contact with parishioners was primarily at the church, whereas contacts in rural settings were most often in the home, on the phone, or in other community-based settings.

Which of the following employment opportunities are among the most dangerous? a. Food processing, textiles, and transportation b. Highway construction, ranching, and public utilities c. Large mechanized factory positions d. Mining, forestry, and fishing

ANS: D Industries with the highest death rates were mining, agriculture, forestry, and fishing, followed by construction, transportation, and public utilities.

A nurse saw there was a need for a hospice facility in a rural area, because so many of the residents were quite elderly and death was common. Which of the following best explains why the community would not immediately see the wisdom of the nurse's suggestion? a. Change is always difficult; the nurse just needs to keep reinforcing the need for such a facility. b. The nurse did not tell residents how much their taxes would need to go up to support such a facility. c. Residents were fearful of confronting death in such an open way. d. Rural residents prefer to use their informal social support networks when help is needed.

ANS: D Limited availability and accessibility of formal health care resources in rural areas combined with self-reliance and self-help traits of rural residents have resulted in the development of strong rural community informal care and social support networks. Rural residents are more apt to entrust care to established informal networks than to new formal care systems. It is unlikely that creation of such a facility would impact their taxes. The nurse needs to partner with the community to implement the change process while building upon the community's strengths, beliefs, and values systems.

external structure

structure that includes the following two categories: (1) extended family, including family of origin and family of procreation; and (2) larger systems, including work, health, and welfare

The mother confided in the nurse, "I don't think my child is improving." Which of the following statements would be the best response by the nurse? a. "My assessment shows your child is improving." b. "The children seem to be playing with and accepting your child." c. "The teacher reported to me that your child is doing much better." d. "What is causing your concern?"

ANS: D Nurses should pay attention, particularly when parents intuitively whisper, "Something is not right." A well-meaning health care provider may attempt to reassure a concerned mother. However, this kind of response may create silence and delay further questions by the parent. Rather than decrease parental concern, it may increase anxiety. The nurse can serve as an intermediary, working among the family and the health care team, to address parental concerns and client goals.

A young woman rolled her wheelchair up the ramp into the clinic to the desk where the intake nurse greeted her. Which of the following should be the nurse's first response? a. "How can I help you?" b. "Who came with you today?" c. Compliment her on her skill with the wheelchair d. Sit so the nurse is on eye level with the young woman

ANS: D Nurses who demonstrate understanding of the issues confronting people with disabilities should approach them on an eye-to-eye level, listening to understand. Moving to eye level should be the first action that the nurse takes before beginning a conversation with the client.

Which of the following would be true for a county to be defined as having persistent poverty? a. At least 10% of the population has been in poverty for at least the last 10 years. b. At least 10% of the population has been in poverty for at least the last 20 years. c. At least 20% of the population has been in poverty for at least the last 20 years. d. At least 20% of the population has been in poverty for at least the last 30 years.

ANS: D Persistent poverty is defined as counties in which 20% of the population has been in poverty over the last 30 years.

From which of the following causes are rural residents twice as likely to die from in comparison with urban residents? a. Chronic diseases, especially cardiac problems and cancers b. Diabetes from the higher incidence of obesity c. Respiratory diseases (from pesticides and herbicides) d. Unintentional injuries

ANS: D Residents of rural areas are nearly twice as likely to die of unintentional injuries, including motor vehicle accidents, when compared with their urban counterparts. The rates of chronic diseases, diabetes, and respiratory diseases are more similar to that of urban residents.

A young man rolled his wheelchair up the ramp into the clinic to the desk where the intake nurse greeted him. After welcoming him to the clinic, which of the following should be the initial question asked by the nurse? a. "Are you a vet home from the war?" b. "Can I help you into the treatment room to see the nurse practitioner?" c. "Do you have any family with you to help today?" d. "What brings you to the clinic today?"

ANS: D The client should be asked what is needed just as any other client would be. A veteran would more probably be at a Veterans Affairs (VA) clinic, and there is nothing in the statement to imply that the clinic is part of the VA medical system. Because the man made it into the clinic, it is doubtful he would need help into the treatment room. Not all persons live with families. The handicap may be unrelated to why he is at the clinic. He may be ill or merely need immunization or other health promotion activity.

When a person was brought into the clinic in a wheelchair, the new nurse asked the patient if he felt a need for a priest. Which of the following best explains why the nurse would ask about spiritual health during the intake assessment? a. All patients have spiritual as well as physical needs. b. Medical personnel have to collaborate with religious leaders. c. People reach out for spiritual care when they have suddenly become ill. d. Some people perceive disabilities as punishment for sin.

ANS: D There are four models for viewing disability including: (1) the medical model, (2) the rehabilitation model, (3) the socially constructed disability model, and (4) the moral model, which connects disability with sin and shame. The nurse apparently perceived disability as a result of sin not yet repented.

Which of the following problems was the Ticket to Work and Work Incentives Improvement Act (TWWIIA) designed to address? a. Employers trying to avoid hiring persons with disabilities b. Employers resenting being forced to hire persons with disabilities c. Persons with disabilities not being able to receive on-the-job training d. Persons with disabilities not seeking employment for fear of losing their health care and other governmental supports

ANS: D Typically, people with disabilities could qualify for such benefits as health care, income assistance programs, and personal care attendant services only if they chose not to work. To address employment and benefit issues for persons with disabilities, in December 1999, the TWWIIA was signed into law. The TWWIIA reduced people with disabilities' disincentives to work by increasing access to vocational services and provided new methods for retaining health insurance after returning to work.

A woman stumbled into the rural health clinic and explained that she had an incredible headache. She was sweating and began vomiting into the wastebasket by the intake nurse's desk. Which of the following diagnoses should be anticipated by the nurse? a. Acute adult panic disorder b. Acute alcohol intoxication c. Acute appendicitis d. Acute meningitis e. Acute pesticide poisoning

ANS: E Signs and symptoms of acute pesticide poisoning are fairly clear, and most health providers in rural communities would recognize them. Common symptoms include headache, dizziness, diaphoresis, nausea, and vomiting. If left untreated, those affected may experience a progression of symptoms including dyspnea, bronchospasm, and muscle twitching. Deaths are relatively uncommon, but they do occur. The nurse is reminded that not all farm laborers are men. The symptoms that are being displayed are not typical of acute panic disorder, acute alcohol intoxication, acute appendicitis, or acute meningitis.

44.) A nurse explained the most recent research about care of the client's condition to a disabled client. The client, rather than express appreciation, said, "I prefer to have my care given this way." The nurse explained that research suggested an alternative approach was more clinically effective, but the client said, "I am more comfortable during the day if my care is given this way." Which of the following actions should be taken by the nurse? Chapter 21 pt. 418.

Accept that the client knows what works best

The landmark civil rights legislation that uses the authority of the federal government to guarantee equal opportunities for people with disabilities related to employment, transportation, public accommodations, public services, and telecommunications is the: Individuals with Disabilities Education Act (IDEA). Ticket to Work and Work Incentives Improvement Act (TWWIIA). Americans with Disabilities Act (ADA). Civil Rights Act.

Americans with Disabilities Act (ADA). The ADA is landmark civil rights legislation that provides a clear and comprehensive mandate against discrimination toward people with disabilities in everyday activities. The IDEA addresses the educational needs of children with disabilities. The TWWIIA reduced people with disabilities' disincentives to work by increasing access to vocational services and provided new methods for retaining health insurance after returning to work. The Civil Rights Act prohibits discrimination based on race, ethnicity, religion, and sex. DIF: Cognitive level: Knowledge REF: Page 414

Based on the Department of Housing and Urban Development (HUD) definitions of homelessness, which one of the following individuals would be considered homeless? An individual whose residence lacks access to public water and electricity An individual who has a permanent nighttime residence in the housing for mentally ill people An individual imprisoned or detained under an act of Congress or state law An individual who spends most nights at public or private places not designed for regular sleeping accommodations

An individual who spends most nights at public or private places not designed for regular sleeping accommodations HUD defines homelessness in four categories: (1) literally homeless, (2) imminent risk of homelessness, (3) homeless under other federal statutes, and (4) fleeing/attempting to flee domestic violence. An individual who spends most nights at public or private places not designed for regular sleeping accommodations is considered literally homeless. Literally homeless is defined as individuals and families who lack a fixed, regular, and adequate nighttime residence and includes a subset for an individual who resided in an emergency shelter or a place not meant for human habitation and who is exiting an institution where he or she temporarily resided. An individual whose residence lacks access to public water and electricity, an individual who has a permanent nighttime residence in the housing for mentally ill people, and an individual imprisoned or detained under an act of Congress or state law are not considered homeless under the HUD definition. DIF: Cognitive level: Knowledge REF: Page 431

A nurse overheard an adult woman, shaking her head in exasperation as she said, "Dad, you're being unreasonable. It's not safe!" Which of the following is the most likely reason for this argument?

Apply warm moist towels to his head, neck, chest, and groin

28.) A woman was in the emergency department with a bruised and broken leg. She explained that when she opened the car door the wind blew it back hitting her leg. Her husband was very attentive, constantly asking the nurse to make his wife more comfortable. Which of the following actions should be taken by the nurse? Chapter 17 pg. 329.

Ask the husband to step out of the room so you can assess the client, and, after he leaves, ask the woman, "Have you ever been deliberately hurt by someone?"

Which of the following are appropriate when working with people with disabilities? (Select all that apply.) Asking a wheelchair-dependent person if he or she needs help and how you can assist. Assuming that a person who is blind knows who is speaking or who is present; otherwise, you might embarrass the individual. Sitting or kneeling at eye level whenever possible when talking with someone who is wheelchair dependent. Speaking distinctly and a little slowly to someone who is hearing impaired. Petting a service dog and complimenting him on what a good boy he is.

Asking a wheelchair-dependent person if he or she needs help and how you can assist. Sitting or kneeling at eye level whenever possible when talking with someone who is wheelchair dependent. Speaking distinctly and a little slowly to someone who is hearing impaired. When working with a blind person, unless the person is quite familiar with everyone present, it is helpful to identify the people present and who is talking during a conversation. Service dogs should not be interrupted when they are working. The other choices are all appropriate ways of interacting with people with disabilities. DIF: Cognitive level: Analysis REF: Page 411

Which of the following statements comparing health risks between men and women is true? A. Men have a higher incidence rate for acute infective and parasitic diseases, digestive system conditions, and respiratory conditions than women. B. Men are at greater risk for higher morbidity and mortality rates for conditions that are the leading causes of death (e.g., heart disease, cancer, stroke) compared with women. C. Men are less likely than women to die from unintentional injuries. D. Men are less likely than women to be involved in criminal activities.

B Men are at greater risk for higher morbidity and mortality rates for conditions that are the leading causes of death (e.g., heart disease, cancer, stroke) compared with women. Men have higher morbidity and mortality rates for conditions that are the leading causes of death. Women have a higher incidence of acute infective and parasitic diseases, digestive conditions, and respiratory conditions than men. Men are more likely to die from unintentional injuries and to be involved in criminal activities.DIF: Cognitive level: KnowledgeREF: Page 341

One of the participants says to Kim, "I am 55 years old. How often should I have my cholesterol checked?" How should Kim respond? A. "You should have your cholesterol checked at least every 6 months regardless of your risk factors." B. "You should have your cholesterol checked at least every year if you have no risk factors." C. "You should have your cholesterol checked at most every 2 years if you have more than five risk factors." D. "You should have your cholesterol checked at least every 5 years dependent on your risk calculations."

B. "You should have your cholesterol checked at least every year if you have no risk factors." You should have your cholesterol checked every year. Blood cholesterol checks for men aged 53 and older are recommended for every year. The optimal interval for screening is uncertain. On the basis of other guidelines and expert opinions, reasonable options include every 5 years, shorter intervals for people who have lipid levels close to those warranting therapy, and longer intervals for those not at increased risk who have had repeatedly normal lipid levels. Screening may be appropriate in older people who have never been screened; repeated screening is less important in older people because lipid levels are less likely to increase after age 65. However, because older adults have an increased baseline risk for coronary heart disease, they stand to gain greater absolute benefit from the treatment of dyslipidemia compared with younger adults. Treatment decisions should take into account a person's overall risk of heart disease rather than lipid levels alone. Overall risk assessment should include the presence and severity of the following risk factors: age, gender, diabetes, elevated blood pressure, family history (in younger adults), and smoking. Risk calculators that incorporate specific information on multiple risk factors provide a more accurate estimation of cardiovascular risk than tools that simply count numbers of risk factors.

Which of the following best describes how the faith community nurse, as a health educator, decides which educational programs to offer the faith community?

Based on the health status and needs of faith community members

33.) Which of the following best describes why grandparents would be the primary caregivers for their grandchildren? Chapter 19 pg. 359 slide#5.

Because the parents were drug abusers or in the prison system

Which of the following best describes why grandparents would be the primary caregivers for their grandchildren?

Because the parents were drug abusers or in the prison system

37.) Which of the following situations best describes when a person would most likely be very unhappy and perhaps ill after retirement? (Select all that apply.) Chapter 19 pg. 359.

If the person's self-image was based on job role and status If the retirement was involuntary If the person has no outside interests or leisure activities

What information should the nursing students share regarding normal physiological changes in the aging population? A. Chronological age is a better measure of age than functional age. B. Urinary incontinence is a normal part of aging. C. The sense of thirst diminishes with aging. D. Polypharmacy is not a concern for older adults.

C. The sense of thirst diminishes with aging. The sense of thirst diminishes with aging. This may lead to dehydration or other nutritional deficits in the aging.

A college student visits the student health center and tells the nurse that his roommate is talking about killing himself, has given some of his possessions away, and he believes the roommate has a gun. Which of the following actions should be taken by the nurse?

Immediately locate the roommate, talk to him, and, if the story is confirmed, stay with him until emergency services arrive.

29.) Which of the following best explains why many elderly women do not prepare meals or do housework? Chapter 17 pg. 323-324.

Chronic diseases, especially arthritis, can make cooking and housework difficult.

Based on the common illnesses and risky health behaviors among farmers, the community health nurse should implement which of the following health promotion activities? Partner with the local health department to offer smoking cessation classes for farmers and their spouses. Collaborate with health care providers and provide support and education for farmers with respiratory conditions and their families. Collaborate with health care providers to conduct skin assessments at a meeting of local farmers. Work with mental health consultations to outline a community-based program to address the high rate of suicide and depression among farmers. Plan, implement, and evaluate an outreach program designed to improve the cardiovascular status of farmers and their families.

Collaborate with health care providers and provide support and education for farmers with respiratory conditions and their families. Several types of farming activities are associated with higher than expected occurrences of acute and chronic respiratory conditions. The role of the nurse is to refer patients to appropriate health care providers and provide support and education for affected people and their families. Farmers have a lower rate of cardiovascular disease, use of mental health services, and tobacco use than their urban cohorts. DIF: Cognitive level: Analysis REF: Page 456

Mr. S was recently hospitalized for a stroke. While in the hospital, he was also diagnosed with diabetes. As his nurse, you know he is at risk for which of the following? Panic disorder Depression Schizophrenia Bipolar disorder

Depression Depression is often a complication of serious physical disorders such as heart attack, stroke, diabetes, and cancer. The other options are usually not complications of other physical disorders. DIF: Cognitive level: Application REF: Page 476

Which of the following statements best describes the demographics of rural America? Despite the shrinking number of family farms and full-time farmers, agriculture continues to be an important part of the rural and U.S. economy. There has been a shift in the economic base of rural America as more communities are dependent on income generated from mining and construction. There has been a decline in diversity in rural America, with more Hispanics and youth moving to urban areas rather than rural communities and small towns. There are more positive health behaviors (e.g., less smoking, obesity, and drug use) among residents of rural America when compared with urban America.

Despite the shrinking number of family farms and full-time farmers, agriculture continues to be an important part of the rural and U.S. economy. Despite the shrinking number of family farms and full-time farmers, agriculture continues to be an important part of the rural and U.S. economy. Changes in rural demographics include a shift in the economic base from agriculture to industry, as well as more diversity as the number of rural Hispanics increases. Additionally, there has been a rise in negative health outcomes for rural residents; obesity, smoking, and drug use are more prevalent in rural America than in urban America. DIF: Cognitive level: Knowledge REF: Page 445

European Federation of Organisations Working with the Homeless (FEANTSA)

Developed a typology of homelessness and housing exclusion that addressed the idea of "home" as having three domains: (1) physical, (2) social, and (3) legal

A nurse is helping a community develop its capacity to address future problems. Which of the following activities is the nurse most likely implementing?

Developing community competence

The community health nurse understands that events such as divorce, remarriage, or separation of parents have the potential to affect the family and individuals within the family. This demonstrates the nurse's understanding of which family theory? Structure-function family theory Family systems theory Family communication theory Developmental theory

Developmental theory Developmental theory is based on stages of family development and family life cycles. A basic premise of developmental theory is the understanding that specific life events, such as separation of parents, divorce, and remarriage, can affect the functioning of each family member and the entire family. Options A, B, and C do not focus on family development and life cycles. DIF: Cognitive level: Knowledge REF: Page 391

A client provides information regarding medical and health history. This is an example of which of the following assessment tools? Genogram Family health tree Ecomap Ancestry mapping tool

Family health tree A family health tree provides a mechanism for recording the family's medical and health histories. The genogram is a tool that helps the nurse outlines the family's structure. The ecomap is used to depict a family's linkages to their suprasystems. Option D is not an assessment tool used by the community health nurse. DIF: Cognitive level: Knowledge REF: Page 392

A client is having difficulty performing his activities of daily living. This means he is having difficulty with which of the following? Expressive functioning Affective functioning Emotional functioning Instrumental functioning

Instrumental functioning Difficulty with activities of daily living is an example of instrumental functioning. Expressive functioning refers to affective or emotional aspects of function. DIF: Cognitive level: Knowledge REF: Page 390

Which of the following is most closely associated with morbidity and mortality outcomes?

Insurance Status

Which of the following statements is true concerning the Individuals with Disabilities Education Act (IDEA)? It is designed to ensure a free, appropriate public education in the most restrictive setting. An IEP (Individualized Education Program) for a child is developed solely by the school professionals. It provides federal funding through state-administered programs from preschool through secondary education for children with disabilities. It removes guidelines that typically result in the termination of Medicaid and Medicare benefits for people with disabilities who return to work.

It provides federal funding through state-administered programs from preschool through secondary education for children with disabilities. This is the only true statement regarding IDEA. IDEA ensures free appropriate public education in the least restrictive setting. The IEP is developed by parents, student, and professionals. The other option concerns the TWWIIA. DIF: Cognitive level: Knowledge REF: Pages 413-414

Which of the following best describes the operational definition of "health literacy" in the United States?

Knowing how to access and use information to make health decisions

What has research suggested is a barrier to college students eating healthy?

Lack of knowledge related to food preparation

A 41-year-old pregnant black client, who is at 35 weeks of gestation, has not seen a physician throughout her pregnancy. Which of the following factors places her most at risk for maternal mortality? Sexually transmitted infections and disorders Lack of prenatal care and advanced maternal age Cultural and ethnic influences Socioeconomic status and urban dwelling

Lack of prenatal care and advanced maternal age Major risk factors for maternal death include lack of antepartal care and family planning services, inadequate health education, and poor nutrition. An additional risk factor, regardless of race, is advancing age. Women age 40 years and older have over three times the risk of dying from a pregnancy-related cause as women age 30 to 39 years. Sexually transmitted infections and disorders and socioeconomic status and urban dwelling may contribute to maternal morbidity. DIF: Cognitive level: Analysis REF: Page 319

Which of the following trends in drug use is being seen among young adults?

Large increase in the use of steroids and methamphetamines

Mr. B has been diagnosed with a severe mental illness. The nurse may expect Mr. B to exhibit which of the following? (Select all that apply.) Independence Low self-esteem Difficulty forming relationships Positive self-image

Low self-esteem Difficulty forming relationships Mentally healthy individuals are independent, have positive self-image, are self-actualized and autonomous, and are able to form meaningful interpersonal relationships. Mental illness, in contrast, refers to maladaptive responses to distress and an inability to mobilize resources. The mentally ill person is often dependent, has low self-esteem, and has difficulty forming interpersonal relationships. DIF: Cognitive level: Knowledge REF: Page 471

Which of the following statements is true? Women experience fewer disabilities resulting from chronic conditions than men do. Most women do not actively seek information that will allow them to be in control of their own health. Many factors that lead to morbidity and mortality among women are preventable. Women report a lower incidence of acute conditions than do males.

Many factors that lead to morbidity and mortality among women are preventable. Most factors that lead to morbidity and mortality among women are preventable. The other statements are incorrect: women are more likely than men to be disabled from chronic conditions, and women experience higher incidence of acute conditions than males. Finally, most women do actively try to control and manage their own health. DIF: Cognitive level: Knowledge REF: Page 316

Agricultural workers are at high risk for occupational injuries and illnesses. To reduce the most common cause of fatalities among youth on farms, which of the following is the most important strategy for the community nurse to implement? Planning a community media campaign advocating the use of helmets Partnering with the county extension agent to offer tractor safety classes Working with local health care providers to encourage use of personal protective equipment, such as hearing and respiratory protection Offering a class for local health care providers addressing the signs and symptoms of pesticide toxicity

Partnering with the county extension agent to offer tractor safety classes Tractor-related accidents, especially rollovers, are the most frequent cause of farm accidents and account for more than one-fourth of farm fatalities. Encouraging the use of helmets is important, but the most important measure would be to offer tractor safety education. Working with local health care providers to encourage use of personal protective equipment, such as hearing and respiratory protection and offering a class for local health care providers addressing the signs and symptoms of pesticide toxicity are less important measures. DIF: Cognitive level: Application REF: Page 456

Which of the following has been confirmed by research regarding the relationship between religion, spirituality, and health?

Persons who attend services have decreased anxiety, depression, and stress.

A nurse is using a macroscopic approach when providing care. Which of the following would the nurse most likely demonstrate?

Providing testimony to legislators concerning proposed legislation

Nutritional Theories

Quality of diet is important, may minimize some of ill effects of the aging process.

A nurse saw there was a need for a hospice facility in a rural area, because so many of the residents were quite elderly and death was common. Which of the following best explains why the community would not immediately see the wisdom of the nurse's suggestion?

Rural residents prefer to use their informal social support networks when help is needed.

Mr. K, age 35 years, is in the nursing clinic for a follow-up health visit. Additional gender-specific educational information should be offered about which of the following subjects? Seat belt usage Falls Digital rectal exams Immunizations

Seat belt usage Men remain three times less likely to use a seat belt as women of the same age despite aggressive legislative action for seat belt usage. The other options are not gender specific. DIF: Cognitive level: Knowledge REF: Page 343

A student athlete receives a screening examination before being active in school sports. Which of the following levels of prevention is being used?

Secondary prevention

An earthquake has hit and buildings are knocked down; people are injured and crying for help. Looters have already started grabbing essential resources. Which of the following actions should the community take first?

Send in law officials to secure the area and ensure the safety of the victims

A new school nurse was concerned about being accepted, as the previous nurse had been at the school for more than 20 years. Which of the following would be the best approach for the nurse to take?

Set up workshops on exercise, nutrition, and weight management and perhaps ongoing blood pressure screenings for school staff

Which of the following best describes how Nightingale responded to challenges about her suggestions for reform of health care

She conveyed her statistical data in more detail and depth, and shared it with political leaders.

24.) The public health nurse is planning an educational program about nutrition to a community group. The nurse plans to include a variety of food choices because the group includes people from wide a range of socioeconomic status. Which of the following groups has the highest poverty rate in the United States? Chapter 17 pg. 321.

Single female heads of household

39.) A young woman rolled her wheelchair up the ramp into the clinic to the desk where the intake nurse greeted her. Which of the following should be the nurse's first response?

Sit so the nurse is at eye level with the young woman

In reviewing research related to client actions that may retard the aging process, the nurse is aware that such research is based on stochastic theories of aging. Why? Stochastic theories state that normal cells divide a specific number of times and the number of cell divisions is proportional to the life span of the species. Stochastic theories state that aging is a programmed decline in the functioning of the nervous, endocrine, and immune systems. Nonstochastic theories propose that aging is a result of a loss of self-regulation between the body and cells, resulting in an immune response that is not reversible or decelerating. Stochastic theories explain aging as the accumulation of random events over time; nonstochastic explain aging as a predetermined process and therefore not modifiable through individual actions. .

Stochastic theories explain aging as the accumulation of random events over time; nonstochastic explain aging as a predetermined process and therefore not modifiable through individual actions. Whereas stochastic theories explain aging as events that occur randomly and accumulate over time, nonstochastic theories view aging as predetermined. Thus, each individual will have a unique accumulation of the cellular processes resulting in aging. Nonstochastic theories describe life as a finite process, with defined cellular life processes, such as cellular division, defining the life span for a given species. Options A and B are examples of nonstochastic theories. DIF: Cognitive level: Knowledge REF: Page 356

Mr. H, an 85-year-old-man, has recently lost his wife. He comes to the local health department to have his blood pressure checked quite often. The nurse should recognize that Mr. H is at greater risk for what? Suicide Homicide Accidental injury HIV infection

Suicide Men are five times more likely than women to commit suicide. Men older than age 85 years are 11 times more likely to die as a result of suicide. Men age 15 to 64 years are two to three times more likely to die as a result of unintentional injury than women of the same age. African American men between ages 45 and 64 years are eight times as likely to die of HIV infection as white men. Among men age 15 to 24 years, African American men are seven times more likely to die of homicide. DIF: Cognitive level: Analysis REF: Page 341

47.) The nurse is talking to a community group about a law that guarantees equal opportunities for people with disabilities related to employment, transportation, public accommodations, public services, and telecommunications. Which law is the nurse talking about? Chapter 21 pg. 413.

The Americans with Disabilities Act (ADA)

Which legislative act prohibits discrimination against pregnant women or conditions involving childbirth or pregnancy? The Affordable Care Act (ACA) The Family and Medical Leave Act (FMLA) The Occupational Safety and Health Act The Civil Rights Act

The Civil Rights Act The Civil Rights Act of 1964 prohibits discrimination based on sex, race, color, religion, or national origin in determining employment or eligibility or termination, wages, and fringe benefits, as well as discrimination against pregnant women or conditions involving childbirth or pregnancy. The ACA, in part, protects women from being denied coverage by insurance companies and being charged more for health care services because of their gender. FMLA allows an employee a minimum provision of 12 weeks unpaid leave each year for family and medical reasons. The Occupational Safety and Health Act helps ensure safe and healthful working conditions for workers. DIF: Cognitive level: Knowledge REF: Page 328

Jane, who is 12 weeks' pregnant, has been having preterm complications with her pregnancy, and her physician has placed her on bed rest. As a result, she will be unable to return to work. Which of the following laws will protect her future employment? The Family and Medical Leave Act (FMLA) Affordable Health Care Act (ACA) Social Security Act Public Health Service Act

The Family and Medical Leave Act (FMLA) FMLA allows an employee a minimum provision of 12 weeks of unpaid leave each year for family and medical reasons such as pregnancy. The FMLA guarantees the employee the same job with the same pay and benefits upon the employee's return to work. The ACA, in part, protects women from being denied coverage by insurance companies and being charged more for health care services because of their gender. The Social Security Act provides monthly retirement and disability benefits to workers and survivor benefits to families of workers covered by Social Security. The Public Health Service provides biomedical and health services research, information dissemination, resource development, technical assistance, and service delivery. DIF: Cognitive level: Knowledge REF: Page 330

Which of the following best describes why risk reduction is of such importance in the United States today?

The leading causes of death are all related to lifestyle choices.

Which statement regarding the impact of managed care on rural public health departments is true? Managed care has expanded the safety-net role of many local health departments by funding primary care services. Medicaid's importance for rural areas is likely to decrease. The role of rural public health departments may increasingly narrow into areas that are currently without any type of reimbursement. Medicaid programs will continue to serve the rural communities on a fee-for-service basis.

The role of rural public health departments may increasingly narrow into areas that are currently without any type of reimbursement. The evolution of managed care into rural environments has limited the safety-net role of some local health departments to provide primary care by preventing fee-for-service reimbursement and contracting care to networks of providers or organizations. This is especially true for Medicaid managed care, which serves that same population of people that are traditionally served with primary care services through local public health departments. Medicaid's importance for rural areas is likely to grow as broader health care developments, such as declining inpatient use of rural hospitals and reductions in Medicare reimbursement, provoke more interest in using the Medicaid system to support threatened rural infrastructure. Consequently, the administration of the Medicaid program will increasingly seek the cost savings promised by managed care, and the role of rural public health departments may increasingly narrow into areas that are currently without any type of reimbursement. DIF: Cognitive level: Knowledge REF: Page 465

Medicare

Title XVIII Social Security Amendment (1965), a federal program administered by the Health Care Financing Administration, which pays specified health care services for all persons older than 65 years who are eligible to receive Social Security benefits

8.) Why does the nurse believe it is important for low-income women to seek prenatal care? Chapter 16 pg. 291, slide# 19.

To obtain referrals for other helpful services.

Non-stochastic Theories

View aging as the result of genetically programmed physiological mechanisms within the body that control the process of aging.

According to the vulnerability index, the individual at highest risk for death is: a 46-year-old person who has been homeless for 2 months and has HIV. a 22-year-old person who lives in a homeless shelter and is addicted to heroin. a 65-year-old person who is being evicted from his home after living there for 25 years. a 60-year-old person who has been homeless for 9 months.

a 60-year-old person who has been homeless for 9 months. Those at high risk for death are individuals who have been homeless for 6 months or more with one or more of the following conditions: More than three hospitalizations or emergency department (ED) visits in 1 year More than 3 ED visits in the previous 3 months 60 years or older Cirrhosis of the liver End-stage renal disease History of frostbite, immersion foot, or hypothermia HIV/AIDS Co-occurring psychiatric, substance abuse, and chronic medical conditions DIF: Cognitive level: Application REF: Page 439

ADA Amendments Act of 2008

a bipartisan bill supported by disability advocates and employers, became effective on January 1, 2009, making it easier for a person "seeking protection under the ADA to establish that he or she has a disability within the meaning of the ADA" (U.S. Department of Justice, 2009)

The U.S. Department of Education defines a homeless child as: a child abandoned in a hospital. a child in foster care. a child in a group home. a child living with a grandparent.

a child abandoned in a hospital. The definition of a homeless child according to the U.S. Department of Education includes children and youth who are: Sharing the housing of other persons because of loss of or inadequate housing Abandoned in hospitals Awaiting foster care placement DIF: Cognitive level: Application REF: Page 432

falls

a condition of coming down freely, usually under the influence of gravity; can be especially harmful for the elderly

handicap

a disadvantage resulting from an impairment or a disability that prevents fulfillment of an expected role

sandwich generation

a family structure in which adults care for elderly parents either in their homes or by providing financial support

anxiety disorders

a group of conditions characterized by feelings of anxiety. Anxiety disorders may be attributed to genetic makeup and life experiences of the individual. Some of the more commonly encountered anxiety disorders are generalized anxiety disorder, panic disorder (sometimes accompanied by agoraphobia), phobias, obsessive-compulsive disorder, and post-traumatic stress disorder (PTSD) (APA, 2000).

metabolic syndrome

a group of risk factors that have been linked to an increased risk of cardiovascular events. These factors include abdominal obesity (waist circumference greater than 35 inches in women), dyslipidemia (elevated triglycerides and low high-density lipoprotein), insulin resistance, and elevated blood pressure.

guardianship

a legal status that places an individual's medical treatment, housing, personal needs, finances, and property in the hands of another person

osteoporosis

a major disorder affecting women that involves loss of bone mass

bipolar disorder

a major mental disorder characterized by episodes of mania, depression, or mixed mood. One or the other phase may be predominant at any given time; one phase may appear alternately with the other, or elements of both phases may be present simultaneously

fetal alcohol spectrum disorder (FASD)

a nervous system dysfunction in infants caused by high consumption of alcohol by pregnant women

A nurse is developing a community education program about how to prevent sudden infant death syndrome (SIDS). Which of the following information would be included in the presentation? (Select all that apply.)

a. Always place a baby on his or her back to sleep. e. Do not let the baby get too hot during sleep. f. Avoid exposing the baby to cigarette or cigar smoke.

Which of the following describe health hazards to which nurses in hospitals are exposed? (Select all that apply.)

a. Anesthetic gases d. Heavy lifting e. Infectious diseases

Children's Health Insurance Program (CHIP)

a program through which states can choose to expand Medicaid eligibility; publicly subsidized health insurance program run by individual states to help close the gap between higher-income and lower-income families whose children need insurance

Early and Periodic Screening, Diagnosis, and Treatment (EPSDT)

a program within Medicaid. It is a comprehensive child health program for uninsured under the age of 21. It includes health education, periodic screening, vision, dental, and hearing services.

elder abuse

a reportable offense of physical, psychological, or material abuse, as well as violation of the rights of safety, security, and adequate health care of older adults

Alzheimer's disease

a slowly progressive brain disorder that begins with mild memory loss and progresses through stages to total incapacitation and eventually death

nuclear family

a small group consisting of parents and their nonadult children living in a single household

pesticide

a substance used to repel pests that may be harmful to ingest or come into contact with

transactional model

a system that focuses on process as opposed to a linear approach

family health tree

a tool based on a genogram that provides a mechanism for recording the family's medical and health histories

When the school health nurse suggested the school health clinic include sexual counseling and free condoms if requested, an angry woman yelled, "Why do I care if some girls have no morals and sleep around?" Which of the following would be the most appropriate response by the nurse?

a. "Because taxpayers spend billions each year for teenage moms and their babies."

Which of the following strategies would be most helpful to implement with adolescent males at a high school? (Select all that apply.)

a. A short required course on how to be a good parent c. Classes in male physiology, self-screenings, and influence of nutrition and exercise f. Interpersonal communication with a male school nurse

A school nurse has identified that there has been an increase in sexually transmitted infections (STIs) among the adolescents attending the school. Which of the following interventions should be completed by the nurse?

a. Arrange for the health department to come to the school to give information and treatment on a confidential basis

Which of the following is the leading cause of disability in the United States?

a. Arthritis

A woman was in the emergency department with a bruised and broken leg. She explained that she had accidentally opened the car door on a trip and the door had blown back on her leg. Her husband was very attentive, constantly asking the nurse to make his wife more comfortable. Which of the following actions should be taken by the nurse?

a. Ask the husband to step out of the room while you get a urine specimen, and, after he leaves, ask the woman, "Have you ever been deliberately hurt by someone?"

Which of the following has been a common result in countries with a severe HIV/AIDS epidemic?

a. Birth rate is high, but life expectancy is only 35 to 40 years.

Which of the following nongovernment organizations (NGOs) is active in improving health on a global scale?

a. Carter Center

Which of the following groups makes up about 30% of the world's population?

a. Children

Which of the following is the most common sexually transmitted disease (STD)?

a. Chlamydia

Which of the following best explains why many elderly women do not prepare meals or do housework?

a. Chronic diseases, especially arthritis and rheumatism, hinder their ability.

Which of the following strategies would most likely increase attendance at a clinic aiming to improve men's health in the community? (Select all that apply.)

a. Employing male nurses b. Having male-friendly space for socialization c. Having evening and weekend hours f. Providing male-accepted and respected activities such as poker

A woman with two young children is debating if she should try to work using child care or stay at home and care for her children. Based on the current trends, which decision is she most likely to make?

a. Enter the work force

Which of the following is a biological explanation for gender differences in health status?

a. Influence of hormones

Which of the following best explains why an employer would care about the health needs of local children?

a. The children are the employer's future workforce.

mental health

absence of mental disorders and the ability for social and occupational functioning

Which of the following describes what has been identified by public health experts as the number one priority in rural areas?

access to health care

instrumental activities of daily living (IADLs)

activities often done in the course of a normal day in a person's life, such as shopping, tracking money or bills, light housework, and meal preparation

activities of daily living (ADLs)

activities usually performed in the course of a normal day in a person's life, such as bathing, dressing, eating, mobility, or toileting

pelvic inflammatory disease (PID)

acute infection of the upper genital tract structures in women, involving any or all of the uterus, fallopian tubes, and ovaries

A woman stumbled into the rural health clinic and explained that she had an incredible headache. She was sweating and began vomiting into the wastebasket by the intake nurse's desk. Which of the following diagnoses should be anticipated by the nurse?

acute pesticide poisioning

ectopic pregnancy

an abnormal pregnancy in which the embryo is implanted outside the uterus

impairment

an anatomical, mental, or psychological loss or another abnormality

agoraphobia

an anxiety disorder characterized by a fear of being in an open, crowded, or public place, where escape is perceived as difficult or help not available in case of sudden incapacitation

obsessive-compulsive disorder (OCD)

an anxiety disorder characterized by recurrent and persistent thoughts, ideas, and feelings of obsessions or compulsions sufficiently severe to cause marked distress, consume considerable time, or significantly interfere with the patient's occupational, social, or interpersonal functioning

family

any person(s) who plays a significant role in an individual's life. This may include a person(s) not legally related to the individual. Members of "family" include spouses, domestic partners, and both different-sex and same-sex significant others. "Family" includes a minor patient's parents, regardless of the gender of either parent...without limitation as encompassing legal parents, foster parents, same-sex parent, step-parents, those serving in loco parentis, and other persons operating in caretaker roles (Human Rights Campaign, 2009, Inclusive Definitions of Family)

The school nurse only had the resources for one education program for all the schools' teachers and support staff. Which of the following topics would be the most important to address in this program?

asthma

Ms. W, 18 years old, presents at a community clinic describing vague complaints. She explains that she recently dropped out of high school and is having trouble keeping a job. She notes that she has difficulty concentrating and has used drugs (i.e., marijuana and alcohol) when riding motorcycles with friends. When conducting the initial assessment, the RN should include questions and tools that would screen for: depression. attention-deficit disorder/attention deficit hyperactivity disorder (ADD/ADHD). posttraumatic stress disorder. obsessive-compulsive disorder.

attention-deficit disorder/attention deficit hyperactivity disorder (ADD/ADHD). The reported symptoms best fit ADD/ADHD, although the young woman should also be screened for depression and substance abuse, which can accompany ADD/ADHD. DIF: Cognitive level: Application REF: Page 479

life expectancy

average years an individual is expected to live

38.) Which of the following concerns of people who have a disability can a nurse help address? Chapter 21 pg. 419 slide# 20.

b. Advocating for removal of environmental and social barriers to needed services

A school nurse at a high school suggests that all students have an educational unit on newborn care in their required health class. Which of the following provides the best rationale for this suggestion?

b. Because the first year of life is the most hazardous until old age

Which of the following nongovernmental groups has been expending millions of dollars to reduce the impact of infectious diseases with profound effect on prevention and treatment for HIV/AIDS, tuberculosis (TB), and malaria?

b. Bill and Melinda Gates Foundation

Which of the following indicators would be most informative in assessing the health of a newborn?

b. Birth weight and length of gestation

Because of county need, a school nurse was transferred from a new suburban school to a large inner-city school in an older low-income neighborhood. Which of the following changes in role should be anticipated by the nurse? (Select all that apply.)

b. Cooperating with the school meal program to be sure fruits and vegetables and other healthy choices are included e. Need for lead poisoning screening and education

Which of the following is the most likely reason for adult women to be injured?

b. Domestic violence

Which of the following are factors that have led to the decrease in mortality of some cancers? (Select all that apply.)

b. Early detection through emphasis on routine screening d. An increased awareness of influence of lifestyle choices f. A decrease in tobacco use and smoking among women

A middle-aged woman was obese and being treated for hypertension; and her blood tests showed elevated triglycerides and low high-density lipoprotein (HDL). Besides recommending physical activity, which of the following nutritional guidance should be provided by the nurse?

b. Eat fish and seafood such as salmon, halibut, tuna, and scallops at least three times a week.

Which of the following is the leading cause of death for women in their first trimester?

b. Ectopic pregnancy

Which of the following groups has the highest poverty rate in the United States?

b. Single female heads of household

Which of the following best explains how marriage customs have remained unchanged over the last couple decades?

b. Women are usually responsible for the children following a divorce.

Which of the following questions would be a good opening to address health concerns with a male client?

c. "What are your usual physical activities?"

Which of the following is the most common cause of death among women after age 75?

c. Cardiovascular disease (CVD)

Which of the following advantages do women have in relation to health screenings? (Select all that apply.)

c. Health care departments focus on maternal and child health. e. Women have more health screenings because of the need for birth control or physician visits related to childbearing.

Which of the following assumptions underlies the Declaration of Alma-Ata?

c. Health is a fundamental human right.

Which of the following statements best describes what tobacco companies have done to counter their losses resulting from U.S. efforts to decrease smoking tobacco?

c. Increased international exports and targeted children

Which of the following statements best describes how the Centers for Disease Control and Prevention (CDC) differs from its many peer agencies?

c. It responds to health emergencies.

When considering the benefits for the recipients, which is more beneficial: Medicaid or private insurance?

c. Medicaid, because it includes early and periodic screening, diagnosis, and treatment services

Which of the following best explains why some community health nurses find nursing models inappropriate for use in their clinical practice?

c. Models, developed for the middle class, do not fit low-income families.

Why do some researchers believe data regarding health behaviors may be inaccurate in relation to men?

c. Most of the data are collected about and from women.

Which of the following statements best describes how the United States compares with other countries on infant mortality rate?

c. Near the bottom of the industrialized nations

2.) Which of the following statements best describes how the United States compares with other countries on infant mortality rate? Chapter 16 slide#14, pg. 288-289.

c. Near the bottom of the industrialized nations.

Using a public health perspective, which of the following has responsibility for a child's health?

c. The entire community

A nurse is trying to determine who would be the most appropriate person to make a health care decision for an adolescent, the adolescent or the parent. Which of the following considerations will the nurse need to make?

c. The need for client autonomy

A school nurse brought a young boy back into a private office and asked him to remove his shirt. Fresh welts across his back were seen. Which of the following would most likely cause the nurse to be concerned about the parents? (Select all that apply.)

c. The parents were dealing with a lot of stress and lacked support. d. The parents were themselves abused as children. e. The parents may be substance abusers.

Preterm birth and low birth weight are the most important predictors of infant health. Factors associated with preterm birth and low birth weight include: (Select all that apply.) age younger than 25 years. chronic stress. poverty. minority status. first pregnancy.

chronic stress. Correct poverty. Correct minority status. Correct Factors associated with preterm birth and low birth weight include minority status, chronic stress, maternal age of younger than 17 years and older than 35 years, chronic health problems, lack of prenatal care, multiple births, certain problems with the uterus or cervix, low socioeconomic status, and unhealthy maternal care involvoing smoking or drug use. DIF: Cognitive level: Knowledge REF: Page 290

"I want to get pregnant as soon as possible now that I'm married," a 43-year-old female says to the nurse. Which of the following would be the most appropriate response by the nurse?

d. "We will help you, but you need to know that there are more risks for you at your age."

How do the policies on children's health in the United States differ from the policies on children's health in other industrialized nations?

d. Although industrialized nations have policies on children's health, the United States does not.

Which of the following actions should be taken by a typical community preparing for the expected demographic changes in the United States?

d. Build more adult condominiums

Which of the following aspects of health care could be learned by a developing country from a developed country?

d. Delivery of primary health care

A male client reports that he is unable to change his health behaviors and adopt a healthier lifestyle. Which of the following is the most likely reason for his difficulty?

d. Lack of motivation

Which of the following factors is the single largest threat to child health?

d. Poverty

"Stand up straight, mom," a young woman said to an older woman in the grocery store checkout line. Which of the following would be the most appropriate action by the nurse who is standing next to them in line?

d. Say, "You both may have some problems. Have you both ever been checked for osteoporosis?"

If most students have no intention of ever leaving their local community, which of the following languages should be chosen as the required foreign language in high school?

d. Spanish, to improve employment opportunities

Which of the following would be the best example of service learning?

d. Student nurses volunteering at the free clinic doing health assessments

Which of the following is most effective in helping males choose healthier lifestyle behaviors?

d. Support from a female family member concerned about the man's well-being

Which of the following best explains why many people decide to move to large urban areas?

d. To seek employment

According to the U. S. Department of Commerce, which of the following rules must be followed by makers of toy guns?

d. Toy guns must be distinguishable from real guns.

A school nurse is developing a health education program for third grade students about how to play safely. Which of the following provides the best rationale for this nursing action?

d. Without proper head protection, many sports can lead to traumatic brain injury (TBI).

Which of the following best explains why lung cancer kills more women than breast cancer?

d. Women have begun smoking.

Which of the following provides the best explanation for the importance of the "Go Red" and "The Heart Truth" campaigns?

d. Women have different symptoms and require different treatment than men.

Which of the following events caused attention that has resulted in the current emphasis on women's health?

d. Women's movement from the 1970s

mortality

death rate, which reflects the number of deaths per unit of population

All of the following contribute to obesity among lower income communities except: the high cost of healthy foods. food insecurity. decreased availability of fast food. lack of access to places to exercise.

decreased availability of fast food. Correct The higher cost of healthy foods, food insecurity, and the lack of access to safe places to exercise contribute to obesity in lower income communities. Fast food tends to be cheap and readily available in lower income communities. DIF: Cognitive level: Knowledge REF: Page 296

Which of the following mental illnesses is most frequently diagnosed among adults in the United States?

depression

mental health disparity

disparities in access, availability, and use of mental health services are well-established by race/ethnicity, geography, and socioeconomic status

Which of the following is a major factor in determining which people or groups are most likely to be helpful to a particular individual?

enviromental proximetry

Nurses can help parents and family members adjust to a child with a disability by: withholding some noncritical information about the condition to decrease the impact. establishing a partnership between the parent and health care team. making decisions on behalf of the child and the family. teaching the family to avoid social networks and support groups to refrain from co-dependency.

establishing a partnership between the parent and health care team Nurses can help the parents and family members adjust to a child with a disability by providing information about the child's condition, providing sources of emotional support from social networks and support groups, empowering and enabling the parent for decision making on behalf of the child, and establishing a partnership between the parent and health care team. DIF: Cognitive level: Knowledge REF: Page 419

In preparing a presentation for seniors and supportive family members, the nurse includes safety information. This is in response to the knowledge that the leading cause of death due to injuries for seniors is: fire. falls. heat or cold stress. motor vehicle accidents.

falls. Falls are the number one cause of fractures, hospital admissions for trauma, loss of independence, and injury deaths. Fall reduction is a Healthy People 2020 objective for older adults. Fire, heat or cold stress, and motor vehicle accidents are also serious safety concerns for older adults. DIF: Cognitive level: Knowledge REF: Page 366

expressive functioning

family functional assessment that includes nine categories: (1) emotional communication, (2) verbal communication, (3) nonverbal communication, (4) circular communication, (5) problem solving, (6) roles, (7) influence, (8) beliefs, and (9) alliances and coalitions

Which gender has higher morbidity rates with a higher prevalence of chronic diseases that cause disability and limitation of activities?

females

Supplemental Security Income (SSI)

funded through general tax revenues; to qualify, the person with a disability must have limited income and resources

The Brown family is being evicted from their home and will be moving into a homeless shelter in 3 days. Under the Department of Housing and Urban Development (HUD) definition, the Brown family is: literally homeless. in imminent risk of homelessness. homeless under other federal statutes. fleeing or attempting to flee domestic violence.

in imminent risk of homelessness. Imminent risk of homelessness defines those who will imminently lose their primary nighttime residence. The literally homeless have no fixed, regular, or adequate nighttime residence. Unaccompanied youth and families with children who are defined as homeless under other federal statues but do not fit into the other HUD defined categories of homeless may be defined as homeless under this HUD definition. Finally, individuals and families who are fleeing or attempting to flee domestic violence or other life-threatening conditions that relate to violence against the individual or family are defined as homeless. DIF: Cognitive level: Application REF: Page 431

mental illness

includes all diagnosable mental disorders, those health conditions characterized by alterations in thinking, mood, or behavior associated with distress and/or impaired functioning

The nurse is facilitating a support group for caregivers of parents with dementias. She discusses strategies to ensure safety of the client who wanders. These strategies include: (Select all that apply.) inform the local police of the potential problem. keep current pictures of the client available. consider restraining the client when left alone. place bells on doors. use childproof doorknobs.

inform the local police of the potential problem. keep current pictures of the client available. place bells on doors. use childproof doorknobs. If wandering is a problem, the local police should be informed of the potential problem; current pictures of the individual should be available; and safety measures such as locks, bells placed on doors, or other means to ensure safety should be installed. The client who wanders should always be supervised. DIF: Cognitive level: Knowledge REF: Page 373

Which of the following is a biological explanation for gender differences in health status?

influence of hormones

crisis intervention team (CIT)

law enforcement, advocacy, and mental health professional team programs designed to improve the way law enforcement and the community respond to people experiencing mental health crises

Americans with Disabilities Act (ADA)

law passed in 1990 that provided a mandate against discrimination toward people with disabilities

Mental Health Parity and Addiction Equity Act 2008

law requires health insurance to cover treatment for mental illness on the same terms and conditions as physical illness

disability

involves a restriction or an inability to perform an activity in a normal manner or within the normal range

Cellular division

is threatened by radiation or chemical reaction, with aging molecular structures that are normally separated become bound together (cross-linking). Effects collagen and reduces tissue elasticity.

Which of the following statements best describes how the American Red Cross (ARC) is different from the Federal Emergency Management Agency (FEMA) or the National Incident Management System?

it is a nongovernmental volunteer agency

The number one health concern identified by the majority of rural health care leaders is: lack of health insurance. limited access to health care. increase in communicable diseases. social isolation of rural dwellers.

lack of health insurance. Rural health leaders identified ten priorities for health care in rural America, with access and affordability to care topping the list. Surveys have found rural uninsured people are more likely to have a usual source of care compared with their urban counterparts. An increase in communicable diseases and social isolation of rural dwellers were not listed as a priority health concern identified by rural health leaders. DIF: Cognitive level: Knowledge REF: Pages 448-449

Which of the following best describes how people with mental illness are viewed by the general public?

lazy, weak. and immoral

intellectual disability (ID)

limitations in the ability of a person to function in areas of daily life

Physiological gender differences that may account for increased mortality rates for men include all of the following except: (Select all that apply.) male immune systems are weaker than women's. men have higher levels of stored iron than women. men have fat stores in their abdomen. men's testosterone levels are cardiac protective. men's brain cells die faster than women's. .

male immune systems are weaker than women's. men have higher levels of stored iron than women. men have fat stores in their abdomen. men's brain cells die faster than women's. Physiological gender differences that may account for increased mortality rates include the following: (1) Some evidence supports the hypothesis that men's higher testosterone levels contribute to lower high-density lipoprotein levels. (2) Body fat distribution may also contribute to sex differences in ischemic heart disease risk, specifically the tendency for men in Western countries to accumulate abdominal body fat versus the tendency for women to accumulate fat on the buttocks and thighs. (3) Men's higher levels of stored iron may also contribute to risk for ischemic heart disease. (4) Male immune systems are weaker than women's. (5) During the process of aging, men's brain cells die faster than women's brain cells. This may explain why men are more often hospitalized for serious mental disease. DIF: Cognitive level: Knowledge REF: Page 343

The community health nurse's commitment to health for all requires an increased awareness of men's health issues; however, men are less likely than women to be seen by a community health nurse. This is because: maternal and child care are the major focus of many public health departments. Correct preventative reproductive health care is the only service available for men. community health nurses do not have the competencies to focus on men's health care needs. health department hours are flexible.

maternal and child care are the major focus of many public health departments. Correct Not only is maternal and child health a major focus of many health departments, but neither a medical nor a nursing specialty within a health department routinely exists to specifically address men's health. Preventive reproductive health care (i.e., family planning, prenatal care, and cancer screening) and associated general screening are not routinely available for men. The hours of services offered by health departments usually do not provide convenient access for men. The community health nurse's commitment to health for all requires an increased awareness of men's health issues in their social and cultural context and individual and group action that will improve men's physical, psychological, and social well-being. DIF: Cognitive level: Knowledge REF: Page 348

Many barriers exist that prohibit positive male health behavior. One barrier that may exist for men is illness orientation. This is because: men may be aware of being ill, but they make a conscious decision not to seek health care to avoid being labeled as "sick." men do not have routine reproductive health checkups that include screening. men may be less willing to talk, may not recall health problems, and may lack a health vocabulary. male health concerns are addressed by specialists and generalists who have not received gender-specific training that would enable them to focus on men's health needs.

men may be aware of being ill, but they make a conscious decision not to seek health care to avoid being labeled as "sick." Illness orientation is the ability to note symptoms and take appropriate action. However, the stereotypical view of men as strong and invulnerable is incongruent health promotion. Option B is a barrier health protection behavior. Option C is a barrier to reporting health behavior. Option D is a factor related to medical care access that impedes men's health. DIF: Cognitive level: Analysis REF: Page 344 Awarded 0.0 points out of 1.0 possible points. 8.ID: 2508982013

prenatal care

monitoring and management of patient during pregnancy to prevent complications of pregnancy and promote a healthy outcome for both mother and infant

A 35-year-old woman came to her appointment for her yearly mammogram and Pap smear. Her physician discussed with her the risk factors for developing breast cancer. The following are risk factors for breast cancer: (Select all that apply.) personal or family history of breast cancer. early age of menarche. late age at menopause. fibrocystic breast disease. multiparous.

personal or family history of breast cancer. early age of menarche. late age at menopause. Personal or family history of breast cancer, early age of menarche, late age at menopause, never having children, having first child after the age of 30 years, and aging are all risk factors for breast cancer. Fibrocystic breast disease is not a risk factor for breast cancer. DIF: Cognitive level: Knowledge REF: Page 324

One strategy for the community health nurse in caring for people with disabilities is becoming a powerful advocate. This involves: providing information but allowing the person to make the decision. supporting the person's choice if it does not conflict with the nurse's. taking control of the person's life and health care. speaking on the person's behalf.

providing information but allowing the person to make the decision. The community health nurse's advocacy for the person affected by a disability extends beyond a resource and referral coordinator role or speaking on the other's behalf. People with disabilities want to speak for themselves. They want to be in control of their lives and their health care. The health care providers should provide information and then leave the choice up to the person. The community health nurse's support should continue even when the client or family's health care decisions conflict with the health care provider's recommendations. DIF: Cognitive level: Knowledge REF: Page 420

Despite nursing activities and public awareness campaigns on this topic, many women face barriers to good health throughout their lives. The barriers to good health women face include the following: (Select all that apply.) racism. poverty. violence. living in a suburban area. lack of access to health care.

racism. Correct poverty. Correct violence. Correct lack of access to health care. Correct Unfortunately, many women face barriers to good health throughout their lives, including racism, violence, poverty, and lack of access to health care. Those living in suburban areas and most outer urban areas experience superior access to health care services compared with those living in rural areas and inner cities, especially if they are poor. DIF: Cognitive level: Knowledge REF: Pages 287-288

alternative housing options

refers to the various types of housing that the elderly may choose. These include retirement communities or apartments, continuing care retirement facilities, assisted living facilities, and board and care facilities and homes. Services at these facilities include meal preparation, housekeeping, laundry, and medication administration.

severe emotional disorders (SED)

specific disorders children and adolescents whose emotional and mental disturbance severely limits their development and welfare over a significant period of time and that required a comprehensive and coordinated system of care in order to meet their needs

Diagnostic Statistical Manual 5 (DSM 5)

standardized manual published by the American Psychiatric Association classifying and establishing criteria for all mental health disorders for both children and adults

internal structure

structure of the family that includes five categories: (1) family composition, (2) gender, (3) rank order, (4) subsystem or labeling the subgroups or dyads, and (5) boundaries

Cesarean section

surgically removing a baby from the mother's womb when natural delivery is not possible

Assertive Community Treatment (ACT)

team treatment approach designed to provide comprehensive, community-based psychiatric treatment, rehabilitation, and support to persons with serious and persistent mental illness such as schizophrenia

Which of the following groups is especially at risk for sexually transmitted diseases (STDs)?

teenage girls

A community health nurse is concerned about reducing the fatal injuries to America's rural children. To address this problem, the nurse organizes a community of solution (see Chapter 1). For this group, it would be important to have representatives from all of the following groups or organizations: (Select all that apply.) the Department of Public Highway and Safety. Correct emergency department staff (both doctors and nurses) from local hospitals. Correct teachers and coaches from area schools. Correct area employers. city or county government. Correct

the Department of Public Highway and Safety. emergency department staff (both doctors and nurses) from local hospitals. teachers and coaches from area schools. city or county government. Limited resources are available in rural areas, and more can be accomplished when resources are pooled. Also, to ensure a more successful outcome, all players with a vested interest in the problem should have a voice in the problem-solving approach. This would include public personnel (city and county government, police, teachers) and private groups (RNs and MDs). Less important for looking at children's issues, however, would be private employers. DIF: Cognitive level: Application REF: Page 461

multiple family configurations

the many variations of individuals that make up family structure (e.g., nuclear families, extended family units, single-parent units, families of group marriages, blended family units, adoptive family units, nonlegal heterosexual unions, lesbian family units, and others)

functional activities

the most basic of activities that are done in a person's life on a normal day, such as seeing, hearing, speaking, walking, using stairs, and lifting and carrying items

Keeping in mind the Healthy People 2020 objectives, the community health nurse is aware that education and actions focused on one specific risk factor may result in a significant reduction in childhood deaths. This is because the leading cause of death for all children older than the age of 1 year is: unintentional injuries. drowning. choking. motor-vehicle related injuries.

unintentional injuries. Correct After the first year of life, unintentional injury is the leading cause of death in children. The leading cause of injury death for children younger than 1 year of age is accidental suffocation caused by choking or strangulation. For children younger than 5 years of age, drowning is the leading cause of death. From age 5 to 14 years, motor vehicle-related injuries are the cause of most deaths. DIF: Cognitive level: Knowledge REF: Page 293

In the role of advocate, the community health nurse recognizes that compared with men in the same age range, women age 65 years and older are most at risk for: divorce. widowhood. social isolation. criminal victimization (e.g., robbery, burglary, assault).

widowhood. Elderly women are two to three times more likely to be widowed compared with men of the same age. Relatively small proportions of older men and women are divorced each year. Social isolation and criminal victimization are equal risks for older men and women. DIF: Cognitive level: Knowledge REF: Page 357

In contrast to people who live in urban areas, rural residents frequently describe their health by their ability to: avoid hospitalizations. maintain social connections. perform activities of daily living. work and be productive.

work and be productive. Rural residents generally describe themselves as healthy if they can do their usual work and remain productive. Performing activities of daily living may be one part of being productive. Avoiding hospitalizations and maintaining social connections are the most frequent descriptions of health by rural residents. DIF: Cognitive level: Application REF: Page 458


Related study sets

Perfusion EAQ, Cardiovascular EAQ

View Set

Northwestern Mutual - Life and Health Insurance

View Set

The 25th amendment and the 4 key sections

View Set

TTMIK Q&A Book: Chapter 1 Question 1: Sample Sentences

View Set

Homework 4: Formation of the Solar System

View Set

Relativity — Admin Essentials I

View Set